Vous êtes sur la page 1sur 140

Test Information

Test Name DAMS MD-MS CBT-2(DECEMBER) 2018 Total Questions 300

Test Type Examination Difficulty Level Difficult

210
Total Marks 1200 Duration
minutes

Test Question    Language:- ENGLISH

1. Prokaryotic promoter of transcription situated 35 bp upstream is:

a. TATA box

b. Goldberg hogness box

c. CAAT box

d. TGG box

Solution. (d) TGG box

Answer. d

2. All are true about fibrous proteins except:

a. They have mainly structural functions

b. They have an axial ratio of < 3

c. Elastin is a fibrous protein

d. They have negligible water solubility

Solution. (b) They have an axial ratio of < 3


Answer. b

3. LDL lipoprotein is associated with :

a. Apoprotein A2

b. Apoprotein B100

c. Apoprotein C1

d. Apoprotein E

Solution. (b) Apoprotein B100


· Low-density lipoprotein (LDL) is one of the five major groups of lipoproteins, which in order of
size, largest to smallest, are chylomicrons, VLDL, IDL, LDL, and HDL, that enable transport of
multiple different fat molecules, including cholesterol,  within the water around cells and within
the water-based bloodstream
· Each native LDL particle contains a single apolipoprotein B-100 molecule (Apo B-100, a protein
that has 4536 amino acid residues and a mass of 514 kDa), which circulates the fatty acids,
keeping them soluble in the aqueous environment. In addition, LDL has a highly hydrophobic
core consisting of polyunsaturated fatty acid known as linoleate and about 1500  esterified
cholesterol molecules.

Answer. b

4. cAMP is required for the activation of :

a. Phosphofructokinase

b. Protein kinase

c. Phosphoglucomutase

d. Hexokinase

Solution. (b) Protein kinase

Some actions of  cyclic AMP are summarized below :

It activates “PROTEIN KINASE”.


Cyclic AMP helps in conversion of cholesterol in pregnenolone.
It stimulates 11-beta – hydroxylase for steroidogenesis.
It stimulates tyrosine hydroxylase for steroidogenesis.
It stimulates tyrosine hydroxylase for catecholamine synthesis. 

Answer. b

5. After a 72-hour fast the substance likely to the involved in the formation of energy would be :

a. Muscle glycogen

b. Liver glycogen

c. Aminoacid

d. Acetoacetate

Solution. (d) Acetoacetate

The ketone bodies, acetoacetate and 13-hydroxy butyrate are acids.


In starvation, the dietary supply of glucose is decreased. The increased rate of lipolysis is
to provide alternate source of fuel.
The excess acetyl CoA is convertedto ketone bodies.
The high glucagon – insulin ratio prevailing under conditions of starvation favours
ketogenesis.
The brain derives 75% of energy from ketone bodies under conditions of fasting.
Hyper-emesis in early pregnancyand prolonged labor are other causes for ketosis in
clinical practice.

Answer. d

6. Which among the following is a feature of non competitive inhibition?

a. Increased Vmax

b. Decreased Vmax

c. Increased Km

d. Decreased Km

Solution. (b) Decreased Vmax

A non competitive inhibitor has no effect on Km. but decrease Vmax


Vmax : Maximum velocity 
Km : The substrate concentration at which the enzyme attains half of the V max

Non competitive
Competitive inhibition
inhibition
Acting on Active site May or may not
Structure of
Substrate analogue Unrelated
inhibitor
Inhibition is Reversible Generally irreversible
Excess substrate Inhibition relieved No effect
Km Increased No change
Vmax No change Decreased
Significance Drug action Toxicological
Answer. b

7. Acute phase reactants are all except

a. Fibrinogen

b. Hepcidin

c. Plasminogen

d. Serum amyloid A protein

Solution. (b) Hepcidin

Answer. b

8. The material which is not involved in protein translation in eukaryotes?

a. RNA polymerase

b. Aminoacyl – tRNA

c. Ribosomes

d. Peptidyl transferase

Solution.

(a) RNA polymerase

RNA polymerase enzyme is involved in transcription process, not in translation process.


Aminoacyl - tRNA is needed during the process of translation to carry the amino acid at
the surface of the ribosome for the attachment in the elongating polypeptide chain.
Ribosome is the organelle on which the protein synthesis takes place.

Answer. a

9. If consecutive nucleotides code for an amino acid, how many amino acid can be theoretically
coded by nucleic acid?
a. 4

b. 64

c. 16

d. 256

Solution. (d) 256

If there would have been two nucleotides in each codon the chance of their varied
combination will be 42 i.e. 16, and three nucleotide in each codon means there will
occur 43 i.e. 64 codons are possible.
According to the above calculation, if there occurs 4 nucleotides in a codon, the chance of
their varied combination will be 44 i.e. 256.

So the answer of above question will be (D) 256

Answer. d

10. Normal role of micro RNA is?

a. Gene regulation

b. RNA splicing

c. Translation initiation

d. Conformational change of DNA

Solution. (a) Gene regulation

Micro RNA and si RNA are both involved in negative gene regulation
Difference between miRNA and si RNA :

miRNA si RNA
-  Involved in negative gene -  Involved in negative gene
regulation regulation

-  Small size (21-25 nucleotide -  Small size (21-25 nucleotide


large) large)

-  Derived from single -  Derived from double


stranded RNA stranded RNA

-  miRNA binds the 3’ -  Si RNA bind nonspecific


untranslated region (UTR) of area of the mRNA and this
the mRNA and thus prevents duplex is degraded in the P
the expression of the mRNA bodies

Answer. a

11. For single carbon transfer, which co-enzyme is responsible?

a. Acetyl Coenzyme A

b. Biotin

c. THFA
d. Pyridoxine

Solution. (c) THFA

Ref.:Read the text below

Sol :

Folic Acid

Coenzyme from: Tetrahydro-folic acid (THFA).

Function : Transporterfor single carbon units (-CH3, - Ch2, COOH) for purine, thymidylate
synthesis and methionine synthesis from homocysteine – requires B12.

Deficiency :

Most common vitamin deficiency in humans


Hematopoietic tissue – anemia
Epithelial cells – nutrient absorpt*ion *impaired.

Answer. c

12. In eukaryotes, initiation of translation of m-RNA requires :

a. Aminoacyl t-RNA synthetase

b. GTP

c. elF

d. Ribosomes 40s

Solution. (c) elF


Ref.:Read the text below
Sol :
- The prefix eIF denotes a eukaryotic initiation factor. For example, eIF – 4E is a protein that
binds directly to the 7-methylguanosine cap, whereas eIF-4A is a helicase.
- The difference in initiation mechanism between prokaryotes and eukaryotes is, in part, a
consequence of the difference in RNA processing.
- The 5' end of mRNA is readily available to ribosomes immediately after transcription in
prokaryotes.

Answer. c

13. Which one among the following is not an antioxidant?

a. Selenium

b. Copper

c. Zinc

d. Iron

Solution.

(d) Iron
Iron is not an an�oxidant.
It can catalyse the forma�on of reac�ve oxygen species and cause oxidant damage.
Selenium, copper, zinc are examples of an�oxidants

An�oxidants :
Vitamins : C, E and A (beta carotene) (Think: “ACE”)
Minerals : Selenium, Zinc, Copper (minor)
Others : Cysteine, Glutathione, Caffeine, allopurinol
Transport proteins : Transferring, Ferri�n, Lactoferrin, Ceruloplasmin.

Answer. d

14. Folic acid requirement per day in a normal male is :

a. 200 μgm/day

b. 300 μgm/day

c. 400 μgm/day

d. 500 μgm/day

Solution.  (a)  200 μgm/day

The RDA of free folate is 200 μg/day. In pregnancy, the requirement is increased to
400μg/day and during lactation to 300 μg/day.
Folic Acid Therapy : Therapeutic dose is 1 mg of folic acid per day orally. Folic acid alone
should not be given in macrocytic anemia. Because it may aggravate the neurological
manifestations of B12 deficiency. So folic acid and vitamin B12 are given in
combination to patients.
Toxictiy of Folic Acid : Doses over 1 mg may cause aggravation of vitamin B12 deficiency
and may precipitate nerve damage. Since solubility of folic acid is low, large doses should
not be given parenterally, as there is danger of crystallization in kidney tubules leading to
renal damage.

Answer. a

15. Using written convention, which one of the following sequences is complementary to
TGGCAGCCT?

a. ACCGTCGGA

b. ACCGUCGGA

c. AGGCTGCCA

d. TGGCTCGGA

Solution. (c) AGGCTGCCA

Ref.:Read the text below

Sol :

Since a chain of DNA are polar, with one end having a 5’-OH group and the other end a
3’-OH group, a conventional means for simply communicating linear structure is
necessary.
Convention dictates that base sequences are written in a 5’ to 3’ direction, much like amino
acids are written in an amino end to carboxyl end direction.
Thus, the complement to 5’ – AGCT – 3’ is 5’ – AGCT -3’ when written
conventionally and 3’-TCGA-5’ when written ina truly complementary way.

Answer. c

16. The following statements concerning the abdominal part of the sympathetic trunk are not
true except:

a. It enters the abdomen behind the lateral arcuate ligament

b. The trunk passes in 6 segmentally arranged ganglia

c. All the ganglia receive white rami communicantes

d. Gray rami communicantes are given off to the lumbar spinal nerves

Solution. (d) Gray Rami Communicantes are given off to the lumbar spinal nerves
· The abdominal portion of the sympathetic trunk is situated in front of the vertebral    1

column, along the medial margin of the Psoas major.


· It consists usually of four lumbar ganglia, connected together by interganglionic cords.
It is continuous above with the thoracic portion beneath the medial lumbocostal arch, and below
with the pelvic portion behind the common iliac artery.
· The ganglia are of small size, and placed much nearer the median line than are the thoracic
ganglia.

· Gray rami communicantes pass from all the ganglia to the lumbar spinal nerves. The first and
second, and sometimes the third, lumbar nerves send white rami communicantes to the
corresponding ganglia.
· The rami communicantes are of considerable length, and accompany the lumbar arteries
around the sides of the bodies of the vertebrae, passing beneath the fibrous arches from which
some of the fibers of the Psoas major arise.

Answer. d

17. Association Fibers are white matter fibers which connect :

a. Corresponding areas in two cerebral hemisphere

b. Cerebral cortex with lower level in brain

c. Different cortical areas in the same hemisphere

d. Different cranial nerve nuclei

Solution. (c) Different cortical areas in the same hemisphere

-  Association fibers : Connect different cortical areas in the same hemisphere

-  Projection Fibers : Connect cerebral cortex with lower level in brain (Eg. Brainstem, spinal
cord)

-  Commissural fibers : Connect corresponding areas in the two cerebral hemispheres

Answer. c
18. Distal end of humerus develops from how many centres

a. 2

b. 3

c. 4

d. 5

Solution. (c) 4.

·         centers are :- capitulum with the lateral part of the trochlea, the medial part of the
trochlea, and one for each epicondyle. The centre for the medial epicondyle forms a separate
epiphysis whereas Thecentres for the lateral epicondyle, capitulum and trochlea fuse around
puberty and form the composite epiphysis

·      About the sixteenth or seventeenth year, the lateral epicondyle and both portions of the
articulating surface, having already joined, unite with the body, and at the eighteenth year the
medial epicondyle becomes joined to it.

Answer. c

19. Which nerve lesion could produce the condition where stimulation of right cornea results in
blinking of the left eye but not the right eye?

a. Left trigeminal

b. Left facial

c. Right trigeminal

d. Right facial

Solution. (d) Right facial

-  Summary of blink reflex :

Ipsilateral blink Contralateral blink


Normal Present Present
Facial nerve lesion Absent Present
Trigeminal nerve
Absent Absent
lesion

Answer. d

20. Atrophy of intrinsic muscles of hand, sensory deficit on medial side of forearm and hand, and
diminished radial pulse on turning the head on the affected side could be because of :

a. Carpal tunnel syndrome

b. Cervical rib

c. Enlarged axillary lymph nodes

d. Supracondylar fracture of humerus


Solution. (b) Cervical rib

-  Cervical ribs (present in upto 1% of people) articulate with the C7 vertebra and are clinically
significant because they may compress spinal nerves C8 and T1 or the inferior trunk of the
brachial plexus supplying the upper limb. Tingling and numbness may occur along the medial
border of the forearm. They may also compress the subclavian artery, resulting in ischemic
muscle pain (caused by poor blood supply) in the upper limb.

Answer. b

21. Fibular collateral ligament is the continuation of:

a. Peroneus longus

b. Biceps femoris

c. Semimembranosus

d. Adductormagnus

Solution. (a) Peroneus longus

·      Fibular collateral ligament is a degenerated part of Peroneus Longus muscle.

·      Oblique popliteal ligament is a continuation of semimembranosus muscle. It lies in the floor
of popliteal fossa.

·      The tibial collateral ligament represents the degenerated part of adductor magnus.

·      The sacrotuberous ligament represents the degenerated part of the long head of biceps
femoris.                               

·      Sacrospinous ligament represents the degenerated part of Coccygeus muscle.

Answer. a

22. The bursa communicating with knee joint is:

a. Infrapatellar

b. Prepatellar

c. Suprapatellar

d. Lateral patellar

Solution.

(c) Suprapatellar
· The suprapatellar bursa communicates with the cavity of knee joint.
· Inflammation of infrapatellar bursa is known as Clergymen knee. & prepatellar bursa is known as Housemaids knee.
· The Anserine bursa separates the insertions of Semitendinosus, Sartorius &Gracilis from tibia.

Answer. c

23. Which of the following is usually responsible for the tumor as shown in the pic?
a. Primary Yolk Sac

b. Hypoblast

c. Primitive Streak

d. Notochord

Solution. C.  Primitive Streak

Ref: Read the text below

Sol :Given pic shows sacrococcygeal teratoma:- Sometimes, remnants of the primitive streak
persist in the sacrococcygeal region. These clusters of pluripotent cells proliferate and form
tumors, known as sacrococcygeal teratomas, which commonly contain tissues derived from all
three germ layers

Answer. c

24. A 45-year-old man presents with a loss of taste sensation over the back of his tongue.
There is no history of regurgitation of fluids through the nose while swallowing, nor any change
in his voice or facial asymmetry. Physical examination shows an absence of the gag reflex.
Palatal movements are normal. There is no deviation, wasting, or fasciculations in the tongue.
Size and movements of the tongue are normal. There is no nasal tone in his speech.
What neural structure is likely to be affected?

a. Vagus nerve

b. Facial nerve

c. Hypoglossal nerve

d. Glossopharyngeal nerve

Solution. d. Glossopharyngeal nerve

Glossopharyngeal nerve is afferent and Vagus nerve is efferent for gag reflex. In given question
Gag reflex is absent so either of IX & X or both are injured.There is no history of regurgitation of
fluids through the nose while swallowing, nor any change in his voice andPalatal movements are
normal, this rules out X nerve injury. No deviation, wasting, or fasciculations in the tongue, this
rules out XII nerve injury. no change in facial asymmetry,this rules out VII nerve injury.

Answer. d

25. The mammary glands are modified versions of which type of gland?
a. Sebaceous gland

b. Lymph gland

c. Sweat gland

d. Endocrine gland

Solution. (c) Sweat gland


· The mammary glands are modified sweat glands.

Answer. c

26. A 23-year-old woman wearing high-heeled shoes inverts and sprainsher ankle while running
down a flight of stairs.
Which of the following ligaments did she most likely injure?

a. Anterior talotibial

b. Calcaneofibular

c. Calcaneotibial

d. Deltoid

Solution. (B) Calcaneofibular

-  Ankle was sprained during inversion so there will be injury on lateral aspect of
ankle.

-  CalcaneofibularLig. Is on lateral aspect of ankle whereas Anterior talotibial,


Calcaneotibial and Deltoid are present on medial aspect of ankle

Answer. b

27. The musculophrenic arteries give rise to ?

a. Anterior intercostal arteries for intercostal spaces 7–9

b. Posterior intercostal arteries for intercostal spaces 3–11

c. Subcostal artery

d. Inferior phrenic artery

Solution. (a) Anterior intercostal arteries for intercostals spaces 7–9

·         The internal thoracic artery gives rise to anterior intercostal arteries for intercostal
spaces 1 – 6.

·         The musculophrenic artery gives rise to anterior intercostal arteries for intercostal spaces
7–9.

Answer. a

28. A 35-year-old woman presents with abdominal tenderness and acute pain. On examination,
her physician observes that an abdominal infection has spread retroperitoneally
Which of the following structures is most likely affected?
a. Stomach

b. Transverse colon

c. Jejunum

d. Descending colon

Solution. (d) Descending colon


· The descending colon is a retroperitoneal organ. The rest of the organs are  surrounded by
peritoneum.

Answer. d

29. The marked structure in the diagram is involved with motor activities. It receives afferents
from all of the following EXCEPT:

a. Spinal cord

b. Thalamus

c. Cerebral cortex

d. Substantia nigra

Solution. a.         Spinal cord

Reference – Read the text below

Sol:   it receives no afferent from Spinal cord whereas Thalamostriate,Nigrostriate and


Corticostriate fibres are afferent to Striatum.

Answer. a

30. A 23-year-old women is brought to the emergency department unconscious after a motor
vehicle accident. The patient was texting on the cell phone & was not wearing a seat belt during
the incident. She has no known medical problems & takes no medications. Her temperature is
36.7°C, blood pressure is 155/90 mmHg, pulse is 62/min & respiratory rate is 10/min. A linear
fracture of skull at the junction of frontal, parietal, temporal & sphenoid bone is seen on head
CT scan. A branch of which of the following arteries is severed in this patient –

a. Facial

b. Maxillary

c. Middle cerebral

d. Occipital

Solution. b Maxillar
a. Exp: point of skull at the junction of frontal, parietal, temporal & sphenoid bone is known as
Pterion. Deep to it lies Middle meningeal artery which is a branch of Maxillary artery.

Answer. b

31. Depolarization phase of the action potential is due to Ca2+  influx in which of the following?

a. Paraspinal muscle

b. Cardiac muscle

c. Extraocular muscle

d. Vascular muscle

Solution. D . Vascular muscle


Ca2+ influx is responsible for depolarization phase of the action potential in pacemaker cells of
the heart and in smooth muscle.

Answer. d

32. All of the following are calcium binding proteins except

a. Calbindin

b. Troponin

c. Calmodulin

d. Calcineurin

Solution. D Calcineurin

Many different Ca2+-binding proteins have been described, including troponin, calmodulin, and
calbindin.
Calbindin-D28k was first shown to be present in the intestine in birds and then found in the
mammalian kidney. It is also expressed in a number of neuronal and endocrine cells, particularly
in the cerebellum. It is encoded in humans by the CALB1 gene.
Calbindin-D28k is a vitamin D responsive gene in many tissues, in particular the chick intestine,
where it has a clear function in mediating calcium absorption.In the brain, its synthesis is
independent of vitamin-D.
Troponin is the Ca2+-binding protein involved in contraction of skeletal muscle.
Calmodulin contains 148 amino acid residues and has four Ca2+-binding domains. It is unique in
that amino acid residue 115 is trimethylated, and it is extensively conserved, being found in
plants as well as animals. When calmodulin binds Ca2+, it is capable of activating five different
calmodulindependent kinases (CaMK), among other proteins. One of the kinases is myosin
light-chain kinase, which phosphorylates myosin. This brings about contraction in smooth
muscle. CaMKI and CaMKII are concerned with synaptic function, and CaMKIII is concerned
with protein synthesis.
Calcineurin is a calmodulin-activated protein. Calcineurin is a phosphatase that inactivates Ca 2+
channels by dephosphorylating them. It also plays a prominent role in activating T cells and is
inhibited by some immunosuppressants.

Answer. d

33. Blood brain barrier is formed by


a. Schwann cells

b. Fibrous astrocytes

c. Protoplasmic astrocytes

d. Tight junctions between endothelial cells of cereberal capillaries

Solution. D   Tight junctions between endothelial cells of cereberal capillaries

Blood brain barrier is formed by tight junctions between the endothelial cells of cerebral
capillariers.

Schwann cells are involved in myelin formation in the peripheral nervous system.

Astrocytes, which are found throughout the brain, are further subdivided into two groups.

Fibrous astrocytes, which contain many intermediate filaments, are found primarily in white
matter.

Protoplasmic astrocytes are found in gray matter and have a granular cytoplasm.

Both types of astrocytes send processes to blood vessels, where they induce capillaries to form
the tight junctions making up the blood–brain barrier. They also send processes that envelop
synapses and the surface of nerve cells. Protoplasmic astrocytes have a membrane potential that
varies with the external K+ concentration but do not generate propagated potentials. They
produce substances that are tropic to neurons, and they help maintain the appropriate
concentration of ions and neurotransmitters by taking up K+ and the neurotransmitters
glutamate and γ-aminobutyrate (GABA).

Answer. d

34. Acetylcholine receptors at the neuro-muscular junctions are

a. Voltage gated sodium channels

b. Ligand gated sodium channles

c. Voltage gated sodium and potassium channels

d. Ligand gated sodium and potassium channels

Solution.

D  Ligand gated sodium and potassium channels

The impulse arriving at the end of the A alpha motor neuron causes release of acetylcholine.
Acetylcholine diffuses to nicotinic cholinergic (NM) receptors that are concentrated at the tops
of the junctional folds of the membrane of the motor endplate. Binding of acetylcholine to these
receptors increases the Na+ and K+ conductance, though the movement of sodium
predominates. The resultant influx of Na+ produces a depolarizing potential, the endplate
potential. The current sink created by this local potential depolarizes the adjacent muscle
membrane to its firing level. Action potentials are generated on either side of the endplate and
are conducted away from the endplate in both directions along the muscle fiber. The muscle
action potential, in turn, initiates muscle contraction. Acetylcholine is then removed from the
synaptic cleft by acetylcholinesterase, which is present in high concentration at the
neuromuscular junction.

Answer. d
35. Identify the correct sequence of the spread of depolarization in the ventricles

a. Septum, endocardium, epicardium

b. Septum, epicardium, endocardium

c. Endocardium, septum, epicardium

d. Endocardium, epicardium, septum

Solution. A Septum, endocardium, epicardium

Depolarization of the ventricular muscle starts at the left side of the interventricular septum and
moves first to the right across the mid portion of the septum. The wave of depolarization then
spreads down the septum to the apex of the heart. It returns along the ventricular walls to the
AV groove, proceeding from the endocardial to the epicardial surface. The last parts of the heart
to be depolarized are the posterobasal portion of the left ventricle, the pulmonary conus, and the
uppermost portion of the septum.

Answer. a

36. Which of the following causes the heart to stop in systole?

a. Hyperkalemia

b. Hypercalcemia

c. Hypernatremia

d. Hypercapnia

Solution. B  Hypercalcemia

Increases in extracellular Ca2+ concentration enhance myocardial contractility. When large


amounts of Ca2+ are infused into experimental animals, the heart relaxes less during diastole
and eventually stops in systole (calcium rigor).However, in clinical conditions associated with
hypercalcemia, the plasma calcium level is rarely if ever high enough to affect the heart.

Hypocalcemia causes prolongation of the ST segment and consequently of the QT interval, a


change that is also produced by phenothiazines and tricyclic antidepressant drugs and by
various diseases of the central nervous system.

As the plasma K+ level rises, the first change in the ECG is the appearance of tall peaked T
waves, a manifestation of altered repolarization. Ventricular arrhythmias may develop. At higher
K+ levels, paralysis of the atria and prolongation of the QRS complexes occur.The resting
membrane potential of the muscle fibers decreases as the extracellular K + concentration
increases. The fibers eventually become unexcitable, and the heart stops in diastole.

Clinically, a fall in the plasma level of Na+ may be associated with low-voltage
electrocardiographic complexes.

Answer. b

37. What is filtration fraction?

a. GFR/ RBF
b. GFR/ RPF

c. RBF/ GFR

d. RPF/ GFR

Solution.

B GFR/ RPF

 Filtration fraction is ratio of the GFR to the RPF. It is normally 0.16–0.20.

The GFR varies less than the RPF. When there is a fall in systemic blood pressure, the GFR falls
less than the RPF because of efferent arteriolar constriction and consequently the filtration
fraction rises.

Answer. b

38. Parathormone increases calcium reabsorption in which of the following segments of the
nephron?

a. PCT

b. TAL

c. DCT

d. CD

Solution. C  DCT

PTH increases calcium absorption in the DCT. But the maximum calcium absorption takes place
in the PCT. PTH decreases absorption of inorganic phosphorus in the PCT (k/a phosphaturic
action of PTH).

Answer. c

39. Maximum work done in quiet breathing is against which of the following?

a. Surface tension elasticity

b. Tissue elasticity

c. Airway resistance

d. Viscous resistance

Solution. A Surface tension elasticity

Work is performed by the respiratory muscles in stretching the elastic tissues of the chest wall
and lungs (elastic work; approximately 65% of the total work), moving inelastic tissues (viscous
resistance; 7% of total), and moving air through the respiratory passages (airway resistance;
28% of total). The dimensions of pressure × volume (g/cm2 × cm3 = g × cm) has the same
dimensions as work (force × distance; g × cm). The amount of elastic work required to inflate
the whole respiratory system is less than the amount required to inflate the lungs alone because
part of the work comes from elastic energy stored in the thorax. Estimates of the total work of
quiet breathing range from 0.3 up to 0.8 kg-m/min. The value rises markedly during exercise,
but the energy cost of breathing in normal individuals represents less than 3% of the total
energy expenditure during exercise. The work of breathing is greatly increased in diseases such
as emphysema, asthma, and heart failure with dyspnea and orthopnea. The respiratory muscles
have length tension relations like those of other skeletal and cardiac muscles, and when they are
severely stretched, they contract with less strength. They can also become fatigued and fail
(pump failure), leading to inadequate ventilation.

Answer. a

40. Surfactant lining the alveoli

a. Helps prevent alveolar collapse.

b. Is produced in alveolar type I cells and secreted into the alveolus.

c. Is increased in the lungs of heavy smokers.

d. Is a glycolipid complex.

Solution. A Helps prevent alveolar collapse.

With expiration as the alveolar size decreases, concentration of surfactant molecules per unit
area increases. The low surface tension when the alveoli are small is due to the presence of
surfactant in the fluid lining the alveoli. Surfactant is a mixture of
dipalmitoylphosphatidylcholine (DPPC), other lipids, and proteins. If the surface tension is not
kept low when the alveoli become smaller during expiration, they collapse in accordance with
the law of Laplace. In spherical structures like an alveolus, the distending pressure (P) equals
two times the tension (T) divided by the radius (r; P = 2T/r); if T is not reduced as r is reduced,
the tension overcomes the distending pressure. Surfactant also helps prevent pulmonary edema.
It has been calculated that if it were not present, the unopposed surface tension in the alveoli
would produce a 20 mm Hg force; such a force would greatly favour transudation of fluid from
the blood into the alveoli.

Answer. a

41. Which of the following is the most alkaline?

a. Saliva

b. Gastric secretions

c. Pancreatic juice

d. Intestinal secretions

Solution. C Pancreatic juice


Pancreatic secretion is the most alkaline with a pH of up to 8.8.

Answer. c

42. Gastic lipase is secreted by which of the following cells?

a. Ebner’s glands

b. Oxyntic cells

c. Chief cells

d. Parietal cells
Solution. C  Chief cells

The most characteristic secretions derive from the glands in the fundus or body of the stomach.
These contain the distinctive parietal cells, which secrete hydrochloric acid and intrinsic factor;
and chief cells, which produce pepsinogens and gastric lipase.

Answer. c

43. The major secretion of the adrenal medulla is which of the following?

a. Nor epinephrine

b. Epinephrine

c. Dopamine

d. Adrenomedulin

Solution. B Epinephrine

Norepinephrine, epinephrine, and small amounts of dopamine are synthesized by the adrenal
medulla. Cats and some other species secrete mainly norepinephrine, but in dogs and humans,
most of the catecholamine output in the adrenal vein is epinephrine. Norepinephrine also enters
the circulation from noradrenergic nerve endings.

In the medulla, epinephrine and nor epinephrine are stored in granules with ATP. The granules
also contain chromogranin A. Secretion is initiated by acetylcholine released from the
preganglionic neurons that innervate the secretory cells. Acetylcholine activates cation channels
allowing Ca2+ to enter the cells from the ECF and trigger the exocytosis of the granules. In this
manner, catecholamines, ATP, and proteins from the granules are all released into the blood
together. Epinephrine-containing cells of the medulla also contain and secrete opioid peptides.
The precursor molecule is preproenkephalin. Most of the circulating metenkephalin comes from
the adrenal medulla. The circulating opioid peptides do not cross the blood–brain barrier.
Adrenomedullin, a vasodepressor polypeptide found in the adrenal medulla.

Answer. b

44. Glucocorticoid secretion is unaffected in

a. Surgery

b. Trauma

c. Haemorrhage

d. High potassium intake

Solution. D High potassium intake


High potassium intake increases aldosterone secretion but glucocorticoid secretion remains
unaffected. Surgery, trauma, stress, haemorrhage increases glocorticoid secretion.

Answer. d

45. Calcitonin is secreted by

a. Thyroid gland

b. Parathyroid gland
c. Thymus

d. Adrenal gland

Solution. A  Thyroid gland

Calcitonin, a calcium-lowering hormone, is secreted primarily by cells in the thyroid gland,


inhibits bone resorption. Although the role of calcitonin seems to be relatively minor, all three
hormones- parathormone, 1,25- dihyrpxycholecalciferol and calcitonin probably operate in
concert to maintain the constancy of the calcium level in the body fluids.

Answer. a

46. The thalamic relay nucleus for taste is

a. Ventroposteromedial

b. Ventroposterolateral

c. Dorsomedial

d. Intralaminar

Solution. A  Ventroposteromedial

VPM is the relay nucleus for taste.


VPL is the relay nucleus for somatic sensations.
Dorsomedial thalamic is for olfaction.
Intralaminar id a non specific nucleus.

Answer. a

47. All are health care delivery indicators except:

a. Doctor population ratio

b. Doctor Nurse ratio

c. Population bed ratio

d. Bed turn-over ratio

Solution. D: Bed turn-over ratio


Answer. d

48. Which of the following is used to compare mortality pattern between 2 populations?

a. Age specific death rates

b. Age standardized death rates

c. Case fatality rate

d. Proportional mortality rate

Solution. B: Age standardized death rates

a.     Age specific death rates- To compare risk of death in two different age groups

b.     Age standardized death rates- To compare morality pattern between 2 populations.
c.      Case fatality rate –Killing power of disease

d.     Proportional mortality rate – Burden of disease (in terms of mortality )

Answer. b

49. All of the following can decrease prevalence except:

a. High case fatality

b. Improved cure rate of cases

c. Inmigration of healthy individuals

d. Outmigration of healthy individuals

Solution.

D: Outmigration of healthy individuals

Factors influencing prevalence


Increased by : Decreased by :

· Longer duration of the disease Shorter duration of the disease


· Prolongation of life of patients High case – fatality rate from
without care disease
· Increase in new cases Decrease in new cases

(Increase in incidence) (Decrease in incidence)


· Out-migration of Healthy people In migration of Healthy people
· In migration of cases Out migration of cases
· In migration of susceptible people Out migration of susceptible people
· Improved diagnostic facilities Improved cure Rate of cases

Answer. d

50. All of the following are anthropozoonoses except:

a. Rabies

b. Plague

c. Anthrax

d. Polio
Solution.

D: Polio

Zoonosis

- Anthropozoonosis - Animal à Man

e.g. Rabies, Plague, Anthrax

- Zooanthrooponosis – Man à Animal

e.g. Human TB in cattles

- Amphixenosis – Man Lower vertebrates

e.g. T.Cruzi , S. japonicum

Answer. d

51. Which of the following is not a criteria for causation?

a. Specificity of association

b. Sensitivity of association

c. Temporal association

d. Dose response relationship

Solution.

B: Sensitivity of association

CRITERIA FOR JUDGING CAUSALITY

Surgeon General of public health in US (Bradford Hill)

1. Temporal association :

Suspected Cause should precede effect. (Most important criteria)

2. Strength of association :
- Large relative risk
- Duration-response relationship
- Cessation (↓ in risk on cessation) in smoking & ca lung
3. Specificity of association :

Most difficult criterion to establish causal association because –

a) single cause can give rise to multiple outcomes


b) Most diseases are due to multiple factors
4. Consistency of the association: Shown by replication of results.
5. Biological plausibility: Strength of association alone done not imply causality, there should be
biological credibility to the association.
6. Coherence of the association
7. Reversibility /Cessation of exposure
8. Judging by evidence
9. Study Design
10. Dose Response Relationship

Answer. b

52. Which of the following is not a primary prevention strategy?

a. Breast self examination

b. Control of tobacco

c. Radiation protection

d. Cancer education

Solution. A: Breast self examination

Primary Prevention-

a)     Actions taken prior to onset of disease which removes the possibility that a disease
will ever  occur

b)    Intervention in pre – pathogenesis phase

c)     risk factors are present but disease has not yet taken place

d)    Modes of intervention

o   Health promotion - Health Education, Environmental modification , lifestyle &


behavioral changes

o   Specific protection – Specific intervention. E.g. Immunization,  chemoprophylaxis,


use of specific nutrient etc

Answer. a

53. “Open vial policy” is not applicable to the following vaccines except:

a. DPT

b. BCG

c. Measles

d. JE vaccine

Solution. A: DPT

Under the UIP, vaccines are supplied in multi-dose packing and can be categorized in two
groups:

Vaccines requiring reconstitution before use: BCG, Measles and JE 

Liquid vaccines which can be administered directly: DPT, Hep-B, OPV, Pentavalent and TT
vaccines

Vaccines requiring reconstitution i.e. BCG, Measles and JE need to be used within 4 hours of
reconstitution.

The remaining vaccines (non-reconstituted, liquid vaccines i.e. DPT, TT, Hep B, OPV and
Pentavalent) retain their ability to be reused within 28 days if maintained under appropriate cold
chain conditions i.e. +2 C to +8 C temperature.

The multi-dose open vial policy (OVP) applies to multi-dose vials of DPT, TT, Hepatitis B, Oral
Polio and Liquid Pentavalent vaccines.

Answer. a

54. Out of total cases, 60% are exposed and out of controls, 20% are exposed. Calculate odds
ratio.

a. 4

b. 6

c. 8

d. 12

Solution.

B: 6

CASES CONTROLS
EXPOSED 60 20
NON EXPOSED 40 80

OR = 60 x 80 / 20 x 40 = 6

Answer. b

55. The incidence of bladder carcinoma is 3 times more in exposed compared to non-exposed.
Based on this, what is the attributable risk in proportion?

a. 40%

b. 66%

c. 50%

d. 80%

Solution. B: 66%

The incidence of bladder carcinoma is 3 times more in exposed compared to non exposed.
So , RR = 3 / 1 = 3

Attributable risk (in proportion) = RR -1 / RR

                                                          = 3-1 / 3

                                                          = 2/3 = 66%

Answer. b

56. The number of patients you need to treat to prevent one additional bad outcome or to
benefit one patient is:

a. Level of significance

b. Number needed to treat

c. Number needed to harm

d. Sample size

Solution. B: Number needed to treat

Number Needed to Treat (NNT)

·        NNT:  is a statistic that summarizes the effectiveness of a therapy, or a preventive
measure, in achieving a desired outcome.

·        It is one way to indicate the clinical significance of an intervention. The simple idea is
that no treatment works for everybody, so how many do you need to treat to benefit one.

·         NNT is the number of patients you need to treat to CURE one patient or to PREVENT
one bad outcome (death, stroke, etc.)

·        Ex:If a drug has an NNT of 5, it means you have to treat 5 people with the drug to
prevent one additional bad outcome OR we should treat 5 patients to CURE one.

·         The ideal NNT would be 1, where all the patients in the treatment group have
improved, but no-one has in the control arm.

Higher the NNT, the less effective is treatment, because more people need to receive the
treatment to see a benefit in one.

Answer. b

57. Period between the possible time of detection and the actual time of diagnosis is :

a. Lead time

b. Screening time

c. Generation time

d. Serial interval

Solution. A: Lead time


Answer. a

58. A test for Hepatitis C is performed for 200 patients with biopsy proven disease and 200
patients known to be free from disease. The test shows positive result in 180 patients with the
disease and negative result in 150 patients without the disease. Which of the following is true?

a. Sensitivity of this test is 80%

b. The proportion of false negatives is 20%

c. Specificity of this test is 80%

d. The proportion of false positives is 25%

Solution. D: The proportion of false positives is 25%

Answer. d

59. A category 1 TB patient on treatment is sputum positive after 2 months of intensive phase.
What is the next line of management ?
a. Extend 1 month of intensive phase

b. Start Cat-II treatment

c. Start Continuation phase

d. Restart cat-I treatment

Solution. C: Start Continuation phase

• There is no provision to extend intensive phase


• Start continuation phase.

Answer. c

60. Which of the following is false regarding Pradhan Mantri Surakshit Matritva Abhiyan
(PMSMA) :

a. The free antenatal check-up will take place on 9th of every month.

b. ANC will take place at government and private hospitals and private clinics across the
country.

c. The scheme is applicable to the pregnant women in second or third trimester.

d. Green sticker on their MCP card indicates high blood pressure.

Solution. D: Green sticker on their MCP card indicates high blood pressure.

Pradhan Mantri Surakshit Matritva Abhiyan (PMSMA) :

·        A minimum package of antenatal care services (including investigations and drugs)
would be provided to the beneficiaries on the 9th day of every month

·        At identified public health facilities (PHCs/ CHCs, DHs/ urban health facilities etc)
in both urban and rural areas in addition to the routine ANC at the health facility/
outreach.

·        Antenatal checkup services would be provided by OBGY specialists /


Radiologist/physicians with support from private sector doctors.

·        It is decided that a minimum package of investigations (including one ultrasound


during the 2nd trimester of pregnancy) and medicines such as IFA supplements,
calcium supplements etc would be provided to all pregnant women attending the
PMSMA clinics.
·        Pregnant women would be given Mother and Child Protection Cards and safe
motherhood booklets.

·        A sticker indicating the condition and risk factor of the pregnant women would be
added onto MCP card for each visit:

          Green Sticker- for women with no risk factor detected

          Red Sticker – for women with high risk pregnancy

          Blue Sticker – For pregnancy induced hypertension

          Yellow Sticker – for co morbidities ( like DM , STDs , hypothyroidism )

Answer. d

61. Which of the following is not true as per recent “Malaria drug policy’ except:

a. ACT is used to treat P.vivax

b. Primaquine is not used to treat P.falciparum along with ACT

c. Mefloquine is used for chemoprophylaxis

d. ACT is used to treat P.ovale

Solution. C: Mefloquine is used for chemoprophylaxis

• ACT is used to treat P.falciparum


• Primaquine is used to treat P.falciparum along with ACT
• Mefloquine is used for long term chemoprophylaxis
• ACT is used to treat P.malariae

Answer. c

62. Which of the following is false regarding supplementation in I-NIPI :

a. Reproductive women : weekly IFA (with 100mg elemental iron)

b. Adolescents : Biweekly IFA (with 100mg elemental iron)

c. Children (5-10 years ) : weekly IFA (with 80mg elemental iron)

d. All of above

Solution. D: All of above

Age Dose
Biweekly : 1 ml IFA syrup
Children (6-59
months) 1 ml contains 20 mg elemental iron and 200 μg folic
acid

Weekly IFA   :  45 mg elemental iron and 400 μg folic


Children (5-10yrs) acid
Pink tablet
Adolescents
(10-19yrs)

Within school: boys Weekly IFA


and girls (WIFS)

60 mg elemental iron and 500 μg folic acid


Out of school : Only
girls (Anganwadi
centres )
Blue tablet

Daily IFA

60 mg elemental iron and 500 μg folic acid

Pregnant
6 months from second trimester and for 6 months
postpartum

Red tablet
Weekly IFA

60 mg elemental iron and 500 μg folic acid


Reproductive
women (20-49 yrs)

Red tablet

Note :Daily 400 μg folic acid is to be taken ( 3


months before pregnancy and 3 months in 1st
trimester )

Answer. d

63. Which of the following is used to study association of risk factor for a rare disease?

a. Cross sectional study

b. Cohort study

c. Case control study

d. RCT
Solution.

C: Case control study

Case Control study Cohort study


1. Proceeds from “effect to cause” Proceeds from “cause to effect”
1. Starts with disease Starts with exposure to suspected cause
2. Fewer no. of subjects Large number of subjects
3. Suitable to rare disease Not for rare
4. Quick research Long follow-up period
5. Yield OR (estimate of RR) Yield RR, AR
6. Yield information about multiple risk Yield information about multiple
factors. outcome
7. Inexpensive Expensive

Answer. c

64. Which of the following is used to represent relationship between two variables?

a. Bar chart

b. Line diagram

c. Histogram

d. Scatter diagram

Solution. D:  Scatter diagram

Scatter diagram

It is used to represent relationship between two quantitative variables .

Used to represent corelation

Answer. d

65. Not true about Chi square test is?

a. Tests the significance of difference between two proportions

b. Tells about presence or absence of an association between two variables

c. Directly measures the strength of association

d. Can be used when more than two groups are to be compared


Solution. C: Directly  measures  the  strength  of association 

Chi  square test:

-Tests   the   significance   of  difference  between  two  proportions 

- Tells  about  presence    or  absence     of  an  association  between  two variables 

-Can  be used  when  more than  two  groups  are to be  compared 

  Strength  of association  : represented by relative risk , Odds ratio etc

Answer. c

66. Which of the following is true for the contingency table given below?

a. Outcome variable in the study is use of antibiotics

b. Chi square test is used to analyze the statistical significance

c. Degree of freedom for this table is 4

d. All of above

Solution. B: Chi square test is used to analyze the statistical significance

Answer. b

67. All are false about p -value except :

a. Is the probability of committing Type - II error .

b. equal to 1- beta .

c. Is the chance that the presence of difference is not concluded when actually there is
difference.

d. When p- value is less than alpha , the result is statistically significant .

Solution. D  : When   p- value   is  less  than  alpha , the result is statistically    significant .

p  -  value   :

-  Is the   probability of committing  Type   - I  error

- Is the chance that the  presence of    difference   is concluded  when  actually  there is  none
-When   p- value   is  less  than  alpha ,  the   result   is    statistically    significant 

Power  : Is equal  to  1- beta

Answer. d

68. The vector shown below can cause?

a. Malaria

b. Kala azar

c. Dengue

d. Plague

Solution. B: Kala azar

Answer. b

69. Which of the following is the minimum standards for Indian foods:

a. PFA standards

b. FSSAI

c. Codex alimentarius

d. AGMARK

Solution. B: FSSAI

·        FSSAI: Food safety and standards authority of India

·        It is the minimum level of food standards for India

·        PFA standards : Not existing now

Answer. b

70. Symbol shown below is used for :


a. Radioactive waste

b. Carcinogenic substance

c. Biohazard substance

d. Cytotoxic waste

Solution. C:  Biohazard substance

Answer. c

71. Abhiyan Indradhanush is implemented under which ministry?

a. MOHFW

b. Ministry of labour and employment

c. MWCD

d. Ministry of home affairs

Solution. B: Ministry of labour and employment

·        It is using coloured bed sheets in ESI hospitals

·        Implemented under Ministry of labour and employment.

Answer. b

72. Which one of the following anti diabetic drug needs dosage adjustment both in renal and
hepatic failure patient?

a. Sitagliptin

b. Vildagliptin

c. Linagliptin

d. Saxagliptin

Solution. B: Vildagliptin

Vildagliptin dose reduction is needed both in liver and kidney disease (Ref KDT,7 th edn,  page
292)
Alogliptin,saxagliptin, sitagliptin, and vildagliptin  are excreted largely  in the urine; lower
dosesshould be given to patients with reduced renal function (Ref- G&G, page1709, 13 th edn)

Linagliptin binds extensively to plasma proteins and is cleared primarily by thehepatobiliary


system, with little renal clearance, so safe in renal failure (Ref- G&G, page1709, 13 th edn)

Answer. b

73. Which one of the following antipsychotic drugs has partial dopaminergic agonistic activity?

a. Clozapine

b. Aripiprazole

c. Olanzapine

d. Risperidone

Solution. B:  Aripiprazole

(Ref KDT,7th edn,  page 441-442)

Clozapine - Mixed 5HT2A–D2 receptor blockade, D4 receptors, α adrenergic blockade., H1


blocking property, anti choliergic

Aripiprazole partial agonist at D2 and

5-HT1A receptor, but antagonist at 5-HT2 receptor

Risperidone- 5-HT2A + dopamine D2 antagonist (192

Olanzapine- blocking multiple

monoaminergic (D2, 5-HT2, α1, α2) as well as

muscarinic and H1 receptors

Answer. b

74. Which one of the following drugs acts by inhibiting thromboxane synthase enzyme?

a. Aspirin

b. Dezoxiben

c. Losartan

d. Vorapaxer

Solution. B: Dezoxiben

Aspirin inhibiting platelet cyclooxygenase (COX) - 1 ,a critical enzyme in biosynthesis of


thromboxane A2

Vorapaxar an orally active protease activated receptor( PAR- 1) antagonist


Dezoxiben inhibits thromboxane synthase enzyme and inhibits thromboxane A2

Losartan a ARB has uricosuric action and , thromboxane A antagonistic action

Answer. b

75. Cabozantinib is useful in

a. Papillary carcinoma of thyroid

b. Medullary carcinoma of thyroid

c. Multiple myeloma

d. Hepato cellular carcinoma

Solution. B: Medullary carcinoma of thyroid

(Ref- G&G, page1551, 13th edn)

vandetanib and cabozantinib are thyroisine kinase inhibitor useful in medullary cancer of thyroid

lenvatinib useful in treating Papillary and Follicular Carcinomas

sorafenib useful in treating RCC, HCC

sunitinib useful in treating RCC, GIST

Answer. b

76. Alemtuzumab targets against

a. CD 20

b. CD 52

c. CD 38

d. CD 33

Solution. B: CD 52

(Ref- G&G, page 2374, 13th edn)

Alemtuzumabtargeting against CD52 antigen., approved for treatingtreatment of B-cell CLL

Ofatumumab, Obinutuzumab targeting against CD20, useful for CLL

Daratumumab targeting against CD38, approved for treatment of patients with MM in


combination with lenalidomide or bortezomib and dexamethasone

Gemtuzumabagainst CD33 useful in AML


Brentuximab vedotin is an anti-CD30 useful in Hodgkin and anaplastic lymphoma

Answer. b

77. Regarding Acetazolamide, effects on urinary excretion all are true except

a. Increases urinary excretion of HCO3

b. Increases urinary excretion of NH4

c. Increases urinary excretion of K

d. Increases urinary excretion Na

Solution. B Increases urinary excretion of NH4

(Ref- G&G, page 832, 13th edn)

rapid rise in urinary HCO-3 excretion ton

inhibition of excretion of NH4

increase excretion of Na, and K

Answer. b

78. Regarding selective alpha 1 blockers, all are true except

a. Terazosin has apoptosis effect on prostate

b. Most doxazosin metabolites are eliminated in the faeces.

c. The chief side effect of silodosin is retrograde ejaculation

d. Prazosin has been found to prolong life in patients with congestive heart failure

Solution. D: Prazosin has been found to prolong life in patients with congestive heart failure

 (Ref- G&G, page 367, 13th edn)

Terazosin and doxazosin induce apoptosis in prostate

Most doxazosin metabolites are eliminated in the faeces.

The chief side effect of silodosin is retrograde ejaculation..

Prazosin has not been found to prolong life in patients with congestive heart failure.

Answer. d

79. Quinidine induced thrombocytopenic purpura is an example for


a. Anaphylactic Reactions

b. Cytolytic Reactions

c. Arthus Reactions

d. Delayed Hypersensitivity Reactions

Solution. B: Cytolytic Reactions

(Ref- G&G, page 93, 13th edn)

penicillin-induced hemolytic anemia, quinidine-induced

thrombocytopenic purpura, and sulfonamide-induced granulocytopenia- all are example for Type
II reaction( Cytolytic Reactions)

Type I: Anaphylactic Reactions

Type III: Arthus Reactions- serum sickness

Type IV: Delayed Hypersensitivity Reactions

Answer. b

80. Selexipag is useful in

a. Systemic hypertension

b. Pulmonary hypertension

c. Erectile dysfunction

d. Prophylaxis of migraine

Solution. B: Pulmonary hypertension

(Ref- G&G, page 1079, 13th edn)

Selexipag - Selective PGI2 receptor agonist- useful in pulmonary hypertension

Oral administration

Side effects include headache, jaw pain, nausea, diarrhea

Answer. b

81. Regarding theophylline, find out the false statement.

a. It produces diuresis due to blocking adenosine receptor

b. High protein diet increases the clearance of theophylline


c. Theophylline inhibits histone de acetylase enzyme

d. Therapeutic range of theophylline is 5- 15mg/L

Solution. C: Theophylline inhibits histone de acetylase enzyme


(Ref- G&G, page 1384, 13th edn)

Increased clearance
Enzyme induction (mainly CYP1A2) by coadministered drugs (e.g., rifampicin, barbiturates,
ethanol)
Smoking (tobacco, marijuana) via CYP1A2 induction
High-protein, low-carbohydrate diet
Barbecued meat
Childhood
Decreased clearance
CYP inhibition (cimetidine, erythromycin, ciprofloxacin, allopurinol, fluvoxamine, zileuton,
zafirlukast)
Congestive heart failure
Liver disease
Pneumonia
Viral infection and vaccination
High-carbohydrate diet
Old age
Theophylline causes Histone deacetylase activation
therapeutic range isnow taken as 5–15 mg/L.

Answer. c

82. Regarding teriparatide, find out the false statement.

a. It is the anabolic agent

b. Increases new bone formation

c. Given orally

d. Approved for both men and women with osteoporosis

Solution. C: Given orally

 (Ref- G&G, page 1758, 13th edn)


Teriparatide- parathormone analogue, it is the anabolicagent,

administered by subcutaneous injection

Approved for both men and women with osteoporosis

abaloparatide, a recently approved synthetic 34-amino acid

analogue of human PTHrP,

Answer. c

83. Not a use of bupropion is

a. Seasonal Depressive disorder

b. Acute psychosis

c. Smoking cessation

d. Obesity

Solution.

D: Obesity

(Ref- G&G, page 462, 13th edn)

Bupropion is indicated for the treatment of depression,

prevention of seasonal depressive disorder, and as a smoking cessation treatment .

Bupropion has effects on sleep EEGs that are opposite those

of most antidepressant drugs.

Bupropion may improve symptoms of ADHD.

used off label for neuropathic pain and weight loss.

Clinically, bupropion iswidely used in combination with SSRIs with the intent of obtaining a greater antidepressant
response.

Answer. d

84. Which one of the following is the CYCLIN-DEPENDENT KINASE(CDK) inhibitior?


a. Palbociclib

b. Panobinostat

c. Pentostatin

d. Ipilimumab

Solution. A: Palbociclib

(Ref- G&G, page 2345,2378, 2294 13th edn)

CDK4/6 inhibitor palbociclib, Abemaciclib and ribociclib were recently approved for the
treatment of breast cancer.

Panobinostat is an orally bioavailable, nonselective pan-HDAC inhibitor, Panobinostat is


approved for the treatment of patients with MM who have received atleast two previous
treatments, including bortezomib and an immunomodulatory agent.

Pentostatin, inhibits adenosine de aminase(ADA), useful in hairy cell leukemia.

Pilimumab- anti-PD-1. Useful in malignant melanoma

Answer. a

85. Venetoclax is approved for the treatment of

a. CLL

b. CML

c. AML

d. ALL

Solution. A:CLL
(Ref- G&G, page 2369 13th edn)
small-molecule inhibitor of BCL2.
USE- CLL

Answer. a

86. All of the following agents will inhibit intestinal epithelial secretion of chloride except

a. Linaclotide

b. Alosetron

c. Crofelemer

d. Budesonide

Solution.  A:Linaclotide
(Ref- G&G, page1807, 13th edn)
Linaclotide will stimulate Cl− and water secretion.
Alosetron inhibit serotonin 5-HT3 receptors
Crofelemer, and budesonide inhibits CL secretion .

Answer. a

87. All of the following drugs causes hyperkalaemia except

a. Trimethoprim

b. Captopril

c. Heparin

d. Amphotericin B

Solution.

D:Amphotericin B

Drugs causing hyperkalaemia-

RAAS blocker

Spironolactone, Eplerenone, Drospirenone, ACE inhibitors

ENaChannel blocker-

Amiloride, triamterene,

Trimethoprim, pentamidine

Nafamostat

NSAIDs

cyclosporine

Succinylcholine

Heparin

Treatment forHyperkalemia

a. Restriction of dietary potassium intake

b. Discontinuation of potassium-sparing diuretics, ACE inhibitors, ARßs,NSAIDs


c. Loop diuretics to promote urinary potassium loss

d. Potassium binding ion-exchange resin (sodium polystyrene sulfonate)

e. Insulin (10 units regular) and glucose (50 mL of 50% dextrose) to

promote entry of potassium intracellularly

f. Inhaled beta-agonist therapy to promote entry of potassium

intracellularly

g. Calcium gluconate or calcium chloride (1 g) to stabilize the

myocardium

Answer. d

88. Co administration of cisplatin with didanosine aggravates which of the following adverse
effects of didanosine?

a. Neuropathy

b. Hepatoxicity

c. Pancreatitis

d. Nephrotoxicity

Solution. A :Neuropathy

(Ref- G&G, page2229, 13th edn)

The most serious toxicities associated with didanosine include peripheral neuropathy and
pancreatitis

Coadministration of other drugs that cause pancreatitis or neuropathy (i.e., stavudine) will
increase the risk and severity of these symptoms.

Ethambutol, isoniazid, vincristine, cisplatin, and pentamidine should be avoided. Serious hepatic
toxicity occurs rarely but can be fatal

 Other reported adverse effects include elevated hepatic transaminases, headache, and
asymptomatic hyperuricemia; noncirrhotic portalhypertension can occur years after exposure to
didanosine.

Answer. a

89. All are p glycoprotein inhibitors except


a. Dronidarone

b. Quinidine

c. Clarithromycin

d. Rifampicin

Solution. D:Rifampicin

 (Ref- harison, page42, 19th edn)

P-glycoprotein inhibitors

Quinidine

Amiodarone, dronidaron

Verapamil

Cyclosporine

Itraconazole

Erythromycin, clarithromycin

Answer. d

90. Isoniazid inhibits all of the following microsomal enzymes except

a. CYP2C19

b. CYP3A4

c. CYP2D6

d. CYP2E1

Solution. D:CYP2E1

 (Ref- G&G, page2099, 13th edn)

Isoniazid is a potent inhibitor of CYP2C19 and CYP3A and a weak inhibitor of CYP2D6 isoniazid
induces CYP2E1
Answer. d

91. Regarding sofosbuvir, all are true except

a. Fat meal increases sofosbuvir’s absorption

b. It is unsafe in decompensated cirrhosis

c. It is not a substrate for CYP enzyme

d. Heart rate monitoring should be done

Solution. B:It is unsafe in decompensated cirrhosis

 (Ref- G&G, page2202, 13th edn)

Sofosbuvir is a prodrug based on a uridine analogue.

Inhibits HCV RNA polymerase.

The dose of sofosbuvir is 400 mg taken once daily.

A high-fat meal increases sofosbuvir’s AUC by 67%–91%.

Sofosbuvir is safe and effective in individualswith decompensated cirrhosis.

Pharmacokinetics are affected by renal impairment, sofosbuvir should not be used in patients
with eGFR of less than 30 mL/min/1.73 m2 or those with ESRD..

Sofosbuvir is active against all HCV.


Sofosbuvir is not a CYP substrate, inhibitor, or inducer and therefore has a low potential for drug
interactions, sofosbuvir is a substrate for

the efflux transporters Pgp and BCRP and should not be used with potent inducers of these
transporters (e.g., rifampin, St. John’s wort, phenytoin,

carbamazepine).

causes bradycardia,  heart rate monitoring needed.

Answer. b

92. The minimum age at which an individual is responsible for his criminal act is

a. 16 years

b. 21 years

c. 7 years

d. 12 years

Solution. C: 7 years

Reference: Section 82 in The Indian Penal Code.

Act of a child under seven years of age.Nothing is an offence which is done by a  child under
seven years of age.

Answer. c

93. Trial of a case takes place in

a. Supreme Court

b. High Court

c. Sessions Court

d. All

Solution. C: Sessions court

Reference: The Essentials of Forensic Medicine & Toxicology, 34th edition, Dr. K.S. Narayan
Reddy, Page No. 7.

Supreme court and High Courts act as Courts of appeal only in criminal cases and do not hold
trial prima facie.

Answer. c

94. Species identification is done by

a. Precipitin test

b. Neutron activation analysis


c. Spectroscopy

d. Benzidine test

Solution. A: Precipitin test

Reference: The Essentials of Forensic Medicine & Toxicology, 34th edition, Dr.K.S. Narayan
Reddy, Page No. 423.

Precipitin test is a species- specific test and is used to differentiate between different species
and humans. Neutron activation analysis is used to identify metals. Spectroscopic examination is
confirmatory test for blood. Benzidine test is a presumptive test for blood.

Answer. a

95. Hydrostatic lung is seen in case of

a. Typical drowning

b. Near drowning

c. Submersion of an unconscious

d. Post-mortem submersion

Solution. D: Post-mortem submersion

Reference: The Essentials of Forensic Medicine & Toxicology, 34th edition, Dr. K.S. Narayan

Reddy, Page No. 347

If a dead body is kept in water at a depth of 2 meters and over a period of 20 hours, (Post-
mortem submersion) ,due to hydrostatic pressure, there will be oedema but without froth,
known as Hydrostatic lung.

Answer. d

96. Which one of the following is the disciplinary action for ‘Professional Misconduct’?

a. Fine

b. Imprisonment

c. Penal erasure

d. Warning notice

Solution. C: Penal erasure

Reference: The Essentials of Forensic Medicine & Toxicology, 34th edition, Dr. K.S. Narayan

Reddy, Page No. 26.

Fine and imprisonment can be passed only by the Court of Law. Warning notice is a code of
conduct issued by MCI or SMC, containing certain practices which are   regarded as falling
within the

meaning of the term “Professional Misconduct”, which is issued to all thendoctors at the time of
registration. It is not a disciplinary action.

Answer. c
97. Declaration of Sydney deals with

a. Terminal illness

b. Brain death

c. Doctor’s role in hunger strike

d. Organ trafficking

Solution.

B: Brain death

Reference: Health related declarations of world medical association.

The Declaration of Istanbul deals with organ trafficking and transplant Tourism.

The Declaration of Malta deals with role of doctors in Hunger strike.

The Declaration of Sydney deals with definition of death (Brain death).

The Declaration of Venice deals with terminal illness.

Answer. b

98. Only tattooing around the wound is seen in

a. Distant shot

b. Intermediate shot

c. Near shot

d. Close shot

Solution. B: Intermediate shot

Reference: The Essentials of Forensic Medicine & Toxicology, 34th edition, Dr.K.S. Narayan

Reddy, Page No. 212, 213.

Intermediate range means the target is beyond the reach of soot and smoke but within the reach
of powder particles, which cause tattooing around the wound of entry on impact. There is no
singeing of hair and blackening in intermediate shot.

Answer. b

99. The phrase ‘Affiliation cases’ refers to

a. Legitimacy

b. Atavism

c. Supposititious child

d. Suits for adoption


Solution. D: Suits for adoption

Reference: The Essentials of Forensic Medicine & Toxicology, 34th edition, Dr. K.S. Narayan

Reddy, Page No. 371.

Affiliation cases (suits for adoption): A woman may allege a particular man to be the father of
her illegitimate child and file a case in the court for fixing the paternity.

As per Sec 125 Cr.P.C., a First class Magistrate can sanction a monthly allowance of any sum
depending upon the circumstances of the case for the maintenance of such child.

Answer. d

100. If the muscles are not stiff in lower limbs, post-mortem hypostasis is not blanching on
pressure, then the post-mortem interval is

a. 8 - 12 hours

b. 2 - 8 hours

c. 8 - 20 hours

d. 6 - 12 hours

Solution. D: 6 - 12 hours

Reference: The Essentials of Forensic Medicine & Toxicology, 34th edition, Dr. K.S. Narayan

Reddy, Page No. 148, 152.

The muscles not stiff in lower limbs means, the rigor mortis has not been fully established. It
takes around 12 hours for full establishment of rigor mortis all over the body. So time since
death is less than 12 hours. The post-mortem hypostasis not blanching on pressure means, it has
fixed. Hypostasis fixes at 6-12 hours. So time since death is more than 6 hours.

Answer. d

101. One of the following is not seen in metal fume fever

a. Fever

b. Chills

c. Malaise

d. Wheezing

Solution. D: Wheezing

Reference: The Essentials of Forensic Medicine & Toxicology, 34th edition, Dr. K.S. Narayan

Reddy, Page No. 511.

All the metals on melting may give rise to a condition called Metal Fume Fever, but it is most
common in case of zinc (Due to inhalation of Zinc oxide). Also known as Monday fever, Foundry
fever, Brass chills, Brazier disease, Smelter shakes. The symptoms are fever, chills and
malaise(resembles with malaria).

Answer. d
102. A 32 year male was clinically diagnosed as a case of Traveller’s diarrhoea. His stool
examination reveals a pear shape structure with falling leaf motility. What is your interpretation?

a. Entamoeba coli

b. Escherichia coli

c. Trichomonas vaginalis

d. Giardia intestinalis

Solution. (d) Giardia intestinalis


· Stool examination shown the trophozoite of Giardia intestinalis.
· Pear shape or leaf shape trophozoite
· Exhibits Falling leaf motility
· MC parasitic cause of Traveller’s diarrhoea
· Treatment : Metronidazole & Tinidazole

Answer. d

103. A 36-year-old male residing in a crowded camp area presents with cough for 4 weeks and
significant weight loss. Which of the following factors is known to be most important in
triggering the granulomatous reaction to wall off and contain the infection?

a. Cord factor

b. Mycolic acid

c. Purified protein derivative (PPD)

d. Wax D

Solution. (a) Cord factor

· Though Cord factor is not a main virulent factor, it helps trigger the Th1 response,  which helps
contain the infection.
· Important virulent factor is Lipoarabinomannan.(LAM)

Answer. a

104. A patient who has been diagnosed with sarcoidosis experiences severe hemoptysis.
Imaging studies are strongly suggestive of bronchiectasis and cavitation. In addition, several
movable masses are detected within the cavitation. Surgical resection of the affected area is
performed and the contents of these cavitary masses are cultured. Microscopic view of the 3-day
culture is shown in the image. Which of the following microbes was isolated?
a. Aspergillus

b. Candida

c. Cryptococcus

d. Pneumocystis

Solution. (a)  Aspergillus

·      Movable cavitary mass and hemoptysis in a patient with a history of sarcoidosis is strongly
suggestive of aspergillomas.

·      Pre-existing conditions, such as tuberculosis, emphysema, or sarcoidosis, which promote


pulmonary cavitation, are linked to the condition. Usually, the infection is non-invasive and may
spontaneously resolve when the underlying condition is corrected.

·      Itraconazole may be indicated, however, if the patient does not prove to be a viable
candidate for surgery.

·      The microscopic structure in the image is a conidiophore characteristic of Aspergillus. By


contrast, Candida and Cryptococcus produce budding yeast cells; and a diagnosis of
Pneumocystis would be supported only if cysts and trophozoites were observed in stained
sections of lung tissue.

Answer. a

105. A chest physician performs bronchoscopy on an HIV positive patient suffering from
pulmonary tuberculosis. To make the bronchoscopy safe for use for next patient, the most
appropriate method to disinfect the endoscope is by :

a. Wipe it with 70% isopropyl alcohol

b. Soak it in 2% Cidex for 30 minutes

c. Autoclaving at 1930C

d. Rinse in 1% sodium hypochlorite for 15 minutes.

Solution. (b)  Soak it in 2% Cidex for 30 minutes

·      Bronchoscopy is heat labile equipment and come under semi critical category of Spaulding
classification.

·      Semi critical category treated with high level disinfectant. Example – 2% Glutaraldehyde
(cidex)

·      Alcohol& Sodium hypochloriteis not effective against M.tuberculosis and endoscopes cannot
tolerate autoclave temperature.

Answer. b
106. Multiple branched lipopolysaccharides in Gram-negative bacteria

a. Inhibit B cells

b. Are thymus independent antigens

c. Are nonantigenic

d. Are haptens

Solution. (b) Are thymus independent antigens

Answer. b

107. Which of the following stains allows differentiation of fungus from human tissue by staining
the fungus a pink-red color?

a. Calcofluor white stain

b. Gomori methenamine-silver stain

c. Periodic acid-Schiff stain

d. Hematoxylin and eosin stain

Solution. (c)  Periodic acid-Schiff stain

·      Calcofluor white stain, Gomori methenamine-silver stain and periodic acid Schiff stain are
all differential stains, but only the periodic acid-Schiff stain turns fungi a pink-red color.

·      The hematoxylin and eosin stain turns fungi a pink-red color also but does not differentiate
between the fungi and human tissue, so it is not a correct answer

Answer. c

108. Foamy liver is caused by

a. Fasciola hepatica

b. Hepatitis G virus

c. Clostridium perfringens

d. Histoplasma capsulatum
Solution. (c)  Clostridium perfringens

·      Pyogenic liver abscess caused by Clostridium perfringens (C. perfringens) is a rare, but
rapidly fatal, infection.

·      Massive haemolysis and gas-forming liver abscess are classical features of this infection
which leads to foamy liver appearance.

Answer. c

109. Calcutta antigen is

a. VDRL antigen

b. O139 Antigen

c. PPD

d. Forsmann antigen

Solution.

(a) VDRL antigen


· VDRL test is a slide flocculation test used widelyfor the diagnosis of syphilis. The test is so named because it was
developed first at the Venereal Disease Research Laboratory,USPHS, New York.
· This is a simple and more rapid test, which uses cardiolipin antigen with added lecithin and cholesterol. This antigen
produced in Serum Institute, Calcutta. So it is commonly called Calcutta Antigen in India.

Answer. a

110. Of the following complement components, which one is the most potent in attracting
neutrophils to the site of infection (i.e., acting as a chemokine)?

a. C1

b. C2

c. C3b

d. C5a

Solution. (d) C5a


· C5a is a very good chemotactic agent for neutrophils.
· C5a also enhances the adhesiveness of neutrophils to the endothelium.

Answer. d

111. Regarding cytomegalovirus (CMV), which one of the following is most accurate?

a. CMV is usually acquired by the faeco-oral route in adults.

b. Neonates born to infected mothers should be given the subunit vaccine.

c. Reactivation of CMV in sensory ganglion cells leads to painful vesicles along nerves.
d. CMV infection of a foetus during the first trimester results in more congenital abnormalities
than infection in the third trimester.

Solution. (d)  CMV infection of a foetus during the first trimester results in more congenital
abnormalities than infection in the third trimester.

·      Congenital abnormalities are more common when a foetus is infected during the first
trimester than later in gestation, because the first trimester is when development of organs
occurs and the death of any precursor cells can result in congenital defects.

·      CMV is transmitted by a variety of modes. Early in life, it is transmitted across the placenta,
within the birth canal, and quite commonly in breast milk.

·      In young children, its most common mode of transmission is via saliva.

·      Later in life it is transmitted sexually; it is present in both semen and cervical secretions.

·      It can also be transmitted during blood transfusions and organ transplants. CMV infection
occurs worldwide, and more than 80% of adults have antibody against this virus.

·      There is no vaccine for CMV.

·      CMV enters a latent state primarily in monocytes and can be reactivated when cell-mediated
immunity is decreased. CMV can also persist in kidneys for years. Reactivation of CMV from the
latent state in cervical cells can result in infection of the newborn during passage through the
birth canal.

Answer. d

112. Your patient is a 75-year-old man who has smoked cigarettes (two packs a day for more
than 50 years) and consumed alcoholic drinks (a pack of 6 beers each day) for most of his adult
life. He now has the signs and symptoms of pneumonia. Gram stain of the sputum reveals
neutrophils but no bacteria. Colonies appear on buffered charcoal yeast (BYCE) agar but not on
blood agar. Which one of the following bacteria is most likely to be the cause of his pneumonia?

a. Bordetella pertussis

b. Haemophilus influenzae

c. Klebsiella pneumoniae

d. Legionella pneumophila

Solution. (d)  Legionella pneumophila

·      The typical candidate for Legionnaires’ disease is an older man who smokes and consumes
substantial amounts of alcohol.

·      Patients with acquired immunodeficiency syndrome (AIDS), cancer, or transplants


(especially renal transplants) or patients being treated with corticosteroids are predisposed to
Legionella pneumonia, which indicates that cell-mediated immunity is the most important
defense mechanism.

·      Sputum Gram stain reveals many neutrophils but no bacteria.

·      The organism fails to grow on ordinary media, but it will grow on charcoal-yeast agar, a
special medium supplemented with iron and cysteine.

Answer. d
113. Regarding haemolytic disease of the newborn (erythroblastosis fetalis), which one of the
following is the most accurate?

a. Maternal red cells are the source of the antigen that induces the antibody.

b. It typically occurs when the father is Rh-positive and the mother is Rh-negative.

c. Maternal IgM anti-Rh antibody enters the fetus and causes damage to the fetal red cells.

d. Symptomatic disease is more likely to occur in the first child than in the subsequent children.

Solution. (b) It typically occurs when the father is Rh-positive and the mother is Rh-negative.

·      Left panel: Fetal red cells (RBCs) bearing the Rh antigen enter the mother’s blood when the
placenta separates during the birth of the first Rh-positive child. IgG antibodies to Rh antigen
are then produced by the mother.

·      Center panel: During a second pregnancy with an Rh-positive fetus, IgG antibodies pass
from the mother into the fetus via the placenta. The antibodies bind to the fetal red cells,
complement is activated, and the membrane attack complex lyses the fetal red cells.

·      Right panel: Anemia and jaundice occur in the fetus/newborn. As a result of the anemia,
large numbers of erythroblasts are produced by the bone marrow and are seen in the blood of
the newborn.

Answer. b

114. Which of the following statements regarding infection with the mumps virus is correct?

a. After initial replication, viremic spread can occur to various organs.

b. Diagnosis is made solely on symptoms, as virus cannot be cultured.

c. Passive immunization is the only means of preventing infection.

d. Reinfection is possible, because of the presence of two viral serotypes.

Solution.

(a) After initial replication, viremic spread can occur to various organs.
· After initial replication in the upper respiratory tract and salivary glands, viral particles are also transmitted to distant
organs such as the kidneys, testes, ovaries, and CNS through viremic spread.
· It can be cultured in lab.
· Active immunization by vaccine prevents disease.
· There is only one serotype.

Answer. a

115. A sexually active 17-year-old man presents to the local free clinic to check some small
papules that appeared on his penis. The papules are small and white and contain a central
depression in their center. There is no penile discharge, nor is there pain on urination. To what
group is the organism most likely associated with?

a. Poxviridae

b. Papovaviridae

c. Adenoviridae

d. Parvoviridae

Solution. (a)  Poxviridae

·      The disease in question is Molluscum contagiosum, which belongs to the Poxviridae and is
characterized by small white papules with a central umbilication usually found in the genital
region; answers B, C and D are incorrect: (B) Papovaviridae include human papillomavirus and
BK, JC polyomavirus, and although HPV causes genital warts, they do not have the central
umbilication present in Molluscum contagiosum; (C) Adenoviridae include a variety of viral
serotypes which cause respiratory, ocular, and gastrointestinal diseases; (D) Parvoviridae include
erythema infectiosum characterized by the slapped cheek appearance.

Answer. a

116. Which of the following is an important distinguishing characteristic of H. pylori as


compared to Campylobacter species?

a. Oxidase production

b. Catalase production

c. Urease production

d. Curved shape

Solution.

(c) Urease production


· Both Campylobacter species and H. pylori have a curved shape, are oxidase and catalase positive, with polar flagellum.
· Urease production is the distinguishing factor of H. pylori, and it is the basis of the urea breath test that diagnoses H.
pylori infection.

Answer. c

117. A foul-smelling specimen was obtained from a 26-year-old woman with a pelvic abscess.
Culture grew both aerobic and anaerobic gram-negative bacteria. The most likely organisms are
which of the following?
a. Actinomyces israelii and Escherichia coli

b. Bacteroides fragilis and Listeria monocytogenes

c. Bacteroides fragilis and Neisseria gonorrhoeae

d. Clostridium perfringens and Bacteroides fragilis

Solution. (c)  Bacteroides fragilisand Neisseria gonorrhoeae

·      In infections, such as intra-abdominal abscesses, Bacteroides species are often associated
with other organisms.

·      The only other organism in the list above that is solely aerobic and gram-negative is N.
gonorrhoeae.

·      Clostridiumand Listeria are both gram-positive.

·      E. coliis gram-negative and a facultative anaerobe.

Answer. c

118. Regarding haptens, which one of the following is the most accurate?

a. A hapten is the antigen-binding site in the hypervariable region of IgG.

b. A hapten cannot induce antibody by itself but can do so when covalently bound to a carrier
protein.

c. A hapten can bind to the antigen receptor of CD4-positive T cells without being processed by
macrophages.

d. A hapten is defined by its ability to bind to the smaller of the two polypeptides that comprise
the class I MHC proteins.

Solution. (b)  A hapten cannot induce antibody by itself but can do so when covalently bound to
a carrier protein.

·      A hapten is a molecule that is not immunogenic by itself  but can react with specific
antibody.

·      Haptens are usually small molecules, but some high-molecular-weight nucleic acids are
haptens as well.

·      Many drugs (e.g., penicillins) are haptens, and the catechol in the plant oil that causes
poison oak and poison ivy is a hapten.

·      Haptens are not immunogenic because they cannot activate helper T cells. The failure of
haptens to activate is due to their inability to bind to MHC proteins; they cannot bind because
they are not polypeptides and only polypeptides can be presented by MHC proteins.
Furthermore, haptens are univalent and therefore cannot activate B cells by themselves.

·      Although haptens cannot stimulate a primary or secondary response by themselves, they
can do so when covalently bound to a “carrier” protein
.

Answer. b

119. A premature infant on intravenous nutrients and high-lipid fluids has developed fungemia
that cultures out on blood agar only when overlaid with sterile olive oil. Microscopy shown in the
image revealed the pathogen. What is the most likely causative agent?

a. Aspergillus

b. Candida

c. Cryptococcus

d. Malassezia

Solution.  (d)  Malassezia

·      Malassezia furfur is a lipophilic fungus that is found primarily in premature infants on
high-lipid intravenous supplements.

·      In LPCB, it is having Spaghetti Meat ball appearance or Banana and Grapes appearance.

Answer. d

120. Weil Felix reaction is useful in diagnosis of

a. Q fever

b. Scrub typhus

c. Trench fever

d. Rickettsial pox.
Solution. (b)  Scrub typhus

·      Weil Felix test is not useful in other conditions.

Weil felix test is used to diagnose typhus fever, tick-borne spotted fever ,scrub fever.

It can not diagnose Brill zinsser’s disease, rickettsial pox and Q fever.Also trench fever can not
be diagnosed by it.

Answer. b

121. A person presents to his physician complaining of chronic GI symptoms. A diagnosis of A.


lumbricoides is made. Human nematodes infect individuals via different routes. This patient is
most likely infected by which of the following?

a. Larvae penetrating unprotected skin

b. Ingesting larvated eggs

c. Eating uncooked pork

d. Internal autoinfection

Solution.  (b) Ingesting larvated eggs

·      Ascaris is acquired by ingesting infective eggs, as are the whipworm, Trichuris trichiura,
and the pinworm, Enterobius vermicularis.

·      Filariform larvae penetrating unprotected skin is the route by which a person becomes
infected by other intestinal nematodes, hookworms and Strongyloides; eating uncooked pork
could lead to trichinosis; and internal autoinfection is caused only by Strongyloides.

Answer. b

122. Following is a histopathological picture from an abdominal mass. The finding is suggestive
of

a. Hyperplasia

b. Dysplasia

c. Metaplasia

d. Anaplasia

Solution. D: Anaplasia
Marked variation in cell and nuclear sizes, the hyperchromatic nuclei, and the presence of
tumour giant cells is suggestive of malignancy and hence the answer is anaplasia.

Answer. d
123. Which of the following chronic inflammatory states can predispose to cancer?

a. Chronic atrophic gastritis

b. Hashimoto thyroiditis

c. Chronic cholecystitis

d. All of the above

Solution. D: All of the above

Answer. d

124. The following gastric condition is associated with

a. CDH1 gene mutation

b. Helicobacter pylori

c. Barret’s oesophagus

d. EBV
Solution. A: CDH1 gene mutation

The histopathology shows a diffuse type of gastric adenocarcinoma is so poorly differentiated


that mostly infiltrating neoplastic cells with marked pleomorphism are seen. Many of the
neoplastic cells have cytoplasm filled with clear vacuoles of mucin , displacing the cell nucleus to
the periphery. This is the “signet ring” cell pattern that is typical of the diffuse type of gastric
adenocarcinoma, which tends to be highly infiltrative and has a poor prognosis. Mutations in
CDH1 that encode E-cadherin involved in epithelial intercellular adhesion are found with some
diffuse gastric carcinomas, including familial forms.

Answer. a

125. All of the following are high risk cytogenetic findings of plasma cell myeloma except

a. t(11;14)

b. deletion 17p

c. t(14;16)

d. t(14;20)

Solution. A: t(11;14)

Answer. a

126. Which of the following combinations is true for myotonic dystrophy?

a. CTG repeats on chromosome 20

b. CTG repeats on chromosome 19

c. CAG repeats on chromosome 20

d. CAG repeats on chromosome 19

Solution. B: CTG repeats on chromosome 19

Answer. b
127. All of the following are false about ER positive Her 2 neu negative breast cancers except

a. Associated with familial BRCA1 mutations

b. Are usually seen in younger women

c. Bone is the most common site of metastasis

d. Early relapse is seen

Solution. C: Bone is the most common site of metastasis

Answer. c

128. Granulomatous inflammation can be seen in all except

a. Takayasu arteritis

b. Churg Strauss Syndrome

c. Polyarteritis nodosa

d. Buerger’s disease

Solution. C: Polyarteritis nodosa

Large vessel vasculitis, Churg Strauss syndrome, Wegener’s granulomatosis and Buerger’s
disease may all show granulomas. Polyarteritis nodosa and Microscopic polyangiitis are
characterised by lack of granulomas.

Answer. c

129. Which of the following are seen in acute hepatitis B?

a. Ductular reaction

b. Ground glass hepatocytes

c. Spotty necrosis

d. Interface hepatitis

Solution. C: Spotty necrosis


Answer. c

130. Juvenile myelomonocytic leukaemia is associated with which inherited syndrome?

a. Down’s syndrome

b. Neurofibromatosis type I

c. Neurofibromatosis type II

d. Turner’s syndrome

Solution. B: Neurofibromatosis type I

Reference: WHO 2017

NF1, the first syndrome found to be associated with a germline mutation in the RAS pathway,
can manifest in early childhood with cafe-au-lait spots, JMML, plexiform neurofibromas, optic
pathway tumours, and bone lesions. In children with NF1, the risk of developing JMML is
estimated to be 200- 350 times the risk in children without the syndrome. For about half of the
patients with JMML and NF1, a positive family history is known .In most affected children, the
clinical diagnosis of NF1 can be made at the time of leukaemic presentation; JMML may be the
first manifestation of NF1 in some of these infants.

Answer. b

131. The distinctive anti-helminthic granule of eosinophils is

a. Eosinophil cationic factor

b. Myeloperoxidase

c. Major basic protein

d. Eosinophil chemotactic factor

Solution. C: Major basic protein

Reference: Harrison’s Principles of Internal Medicine, 20th edition

The distinctive feature of the red-staining (Wright’s stain) eosinophil granule is its crystalline
core consisting of an arginine-rich protein (major basic protein) with histaminase activity,
important in host defence against parasites. Eosinophil granules also contain a unique
eosinophil peroxidase that catalyzes the oxidation of many substances by hydrogen peroxide and
may facilitate killing of microorganisms.

Answer. c

132. Which of the following is/are cause(s) of massive splenomegaly?

a. Chronic myeloid leukaemia

b. Chronic lymphocytic leukaemia

c. Hairy cell leukaemia

d. All of the above

Solution. D: All of the above

Reference: Harrison’s Principles of Internal Medicine, 20th edition

Following are the causes of massive splenomegaly, defined as spleen extending >8 cm below left
costal margin and/or weighing more than 1000 gms-

Answer. d

133. All of the following are true about Crohn’s disease except

a. Terminal ileum is the most common site

b. Earliest lesion is aphthous ulcer

c. Rectal involvement is common

d. Oedema and loss of normal mucosal folds commonly seen

Solution. C: Rectal involvement is common

Crohn’s disease, also known as regional enteritis, may occur in any area of the gastrointestinal
tract but the most common sites involved at presentation are the terminal ileum, ileocecal valve,
and cecum. Disease is limited to the small intestine alone in about 40% of cases; the small
intestine and the colon both are involved in 30% of patients; and the remainder of cases are
characterized by colonic involvement only. Infrequently, Crohn’s disease may involve the
oesophagus or stomach. The presence of multiple, separate, sharply delineated areas of disease,
resulting in skip lesions, is characteristic of Crohn’s disease and may help in differentiation from
ulcerative colitis. Strictures are common. The earliest lesion, the aphthous ulcer, may progress,
andmultiple lesions often coalesce into elongated, serpentine ulcers oriented along the axis of
the bowel. Oedema and loss of normal mucosal folds are common. Sparing of interspersed
mucosa results in a coarsely textured, cobblestone appearance in which diseased tissue is
depressed below the level of normal mucosa.

Answer. c
134. Which of the following is true about Polyarteritis nodosa?

a. Granulomas can be seen

b. Fibrinoid necrosis is present

c. ANCA positivity is common

d. Glomerulonephritis is the most common manifestation

Solution. B: Fibrinoid necrosis is present

PAN is characterised by transmural necrotising inflammation, with no granulomas. ANCA


positivity is rare and it is characterised by arteritis without glomerulonephritis.

Answer. b

135. A 25 years old male patient with painless left cervical lymphadenopathy, night sweats,
significant weight loss, plethora and facial oedema is being investigated. The biopsy from the
enlarged lymph node shows atypically large cells, mononuclear mixed with binucleate cells.
Some of these cells appear to have their nuclei in empty spaces. The immunophenotype of these
cells on IHC is expected to be-

a. CD 45 + CD 30 – CD 15 –

b. CD 45 – CD 30 + CD 15 +

c. CD 45 –CD 30 – CD 15 –

d. CD 45 + CD 30 + CD 15 +

Solution. B: CD 45 – CD 30 + CD 15 +

The clinical history and histopathology is suggestive of Nodular sclerosis type of Classical
Hodgkin lymphoma. Immunophenotype of option B is most suitable.

Answer. b

136. Which of the following is the least commonly involved artery in Takayasu arteritis?

a. Subclavian

b. Pulmonary

c. Renal artery

d. Vertebral artery

Solution. B: Pulmonary
Reference: Harrison’s Principles of Internal Medicine, 20th edition
Answer. b

137. The most important source of histamine:

a. Mast cells

b. Eosinophil

c. Neutrophil

d. Macrophages

Solution. A: Mast cells

Histamine

·         It is formed from the amino acid ‘histidine’.

·         Mast cells are the richest source of histamine.

·         It is also present in platelets and basophils.

·         It causes vasodilation and increased permeability (immediate transient response) and
bronchoconstriction.

Answer. a

138. Wermer syndrome is due to mutation in which of the following

a. DNA helicase

b. DNA topoisomerase

c. MENIN
d. RET

Solution. C: MENIN

Ref. Robbins pathology 9th edition. Chapter 1 and Chapter 24

There are two syndromes which will confuse you

Werner and Wermer syndrome.

Just remember this, Wer”M”er = “M”EN syndrome

Werner – Premature aging syndrome

·         DNA Helicase defect

·         It's a dramatic, rapid appearance of features associated with normal aging. 

·         Individuals with this disorder typically grow and develop normally until they
reach puberty

Wermer syndrome – MEN 1

·         Parathyroid hyperplasia/adenoma

·         Pancreatic islet cell hyperplasia/adenoma/ carcinoma

·         Pituitary hyperplasia/adenoma

·         Mutant gene is MEN 1  (MENIN)

Answer. c

139. A young female had a surgery and post operatively she was found to have the below lesion.
She complains of pain and itching. What type of collagen one would see predominantly in the
below lesion?

a. Collagen 1

b. Collagen 2

c. Collagen 3

d. Collagen 4

Solution. A: Collagen 1

·         The diagnosis is Keloid – owing to the nature of lesion outgrowing the margins of the
injury and also complaints of pain and itch. Both points towards the diagnosis of Keloid.

·         Keloid and hypertrophic scar the predominant collagen is Type 1


Keloid

·         Abnormal fibroblast reaction to injury

·         Usually in people of African descent, often in earlobe

·         High rate of recurrence after excision

Microscopy

·         Wide bands of collagen with large, brightly eosinophilic, glassy fibers

·         Also parallel fibroblasts and myofibroblasts

Answer. a

140. What is the defect in the person with the below finding in peripheral smear?

a. Hb Beta chain mutation – 6th position Glutamine to Valine

b. Defect in enzyme glucose -6- PO4 dehydrogenase

c. Absence of Beta chain of Hb

d. Absence of Alpha chain of Hb

Solution. B: Defect in enzyme glucose -6- PO4 dehydrogenase

The above picture shows bite cells. It’s a clue for the diagnosis of G6PD deficiency

G6PD deficiency

·         G6PD deficiency causes both episodic intravascular and extravascular


hemolysis.

·         On exposure to oxidant stress the amount of free radicals produced is beyond
the capacity of RBC’s to remove them, resulting in free radical mediated damage to
the RBC’s

·         Globin chains, which become denatured and form membrane-bound


precipitates known as Heinz bodies.

·         The membrane of RBC’s is not stained in the part of the Hb coagulum formed
resulting in the formation of bite cells
Answer. b

141. Incomplete penetrance is seen in which of the following disorders?

a. Fragile X syndrome

b. Hemophilia

c. MELAS

d. Marfan’s syndrome

Solution. D: Marfan’s syndrome

Incomplete penetrance is a feature of dominant disorders. Of the four options only Marfan’s is
an autosomal dominant disorder.

Patterns of inheritance of the options

a.       Fragile X syndrome - XLR

b.      Hemophilia  - XLR

c.       MELAS – mitochondrial

d.      Marfans syndrome - AD

Incomplete penetrance and variable expressivity are features of dominant disorders.

Answer. d

142. Stellate cells and loose matrix are features of which the following

a. Rhabdomyoma of heart

b. SCC

c. Myxoma of heart

d. Cat scratch disease

Solution. C:Myxoma of heart

Myxoma of heart

·         Myomas are the most common primary tumor of the heart in adults

·         They are benign neoplasms thought to arise from primitive multipotent
mesenchymal cells.

·         Most common site is Left atrium

·         Myxomas are gelatinous tumors as the name says they have abundance of
myxoid matrix

·         Histologically, myxomas are composed of stellate or globular myxoma cells


embedded within an abundant acid mucopoly- saccharide ground substance

Rhabdomyoma - histologic processing often artifactually reduces the abundant cytoplasm to thin
strands that stretch from the nucleus to the surface membrane, an appearance referred to as
“spider” cells

Answer. c

143. Oncocytic cells in thyroid are diagnostic of

a. Papillary carcinoma thyroid

b. Hashimoto thyroiditis

c. Riedel thyroiditis

d. Medullary carcinoma thyroid

Solution. B: Hashimoto thyroiditis

Hurthle cell is an example for oncocytic cell. Now the question needs no explanation. Answer is
hashimoto thyroiditis

Hashimoto Thyroiditis - Microscopy

·         There is extensive infiltration of the parenchyma by a mononuclear


inflammatory infiltrate containing small lymphocytes, plasma cells, and
well-developed germinal centers

·         The thyroid follicles are atrophic

·         Presence of abundant eosinophilic, granular cytoplasm, termed Hürthle cells.

·         Hurthle cells are -  metaplastic response of the normally low cuboidal follicular
epithelium to ongoing injury

Answer. b

144. Flexner-Wintersteiner rosettes are seen in

a. Neuroblastoma

b. Medulloblastoma

c. Retinoblastoma

d. Ependymoma

Solution. C: Retinoblastoma

Rosettes are little round groupings of cells found in tumors. They usually consist of cells in a
spoke-wheel or halo arrangement surrounding a central, acellular region
Flexner-Wintersteiner Rosettes

·         They are characteristic of retinoblastomas

·         They consist of tumor cells surrounding a central lumen that contains cytoplasmic
extensions from the tumor cells.

·         Tumor cells under electron microscopy, they have features of primitive photoreceptor cells

Answer. c

145. Which is the most common type of pemphigus?

a. Pemphigus vegetans

b. Pemphigus foliaceus

c. Pemphigus vulgaris

d. Pemphigus erythematosus

Solution.

C:Pemphigus vulgaris

Pemphigus vulgaris, by far the most common type

· 80% of all pemphigus

· Oral mucosa; also scalp, face, eye, pharynx, larynx, axilla, groin, trunk; nail involvement

· Fatal if untreated because oral erosions impair swallowing

· Complications are due to staphylococcal infection

Microscopy

· Numerous small, flaccid, suprabasilar bullae with single row of keratinocytes attached to
basement membrane

· Prominent extension of acantholysis into follicular infundibula

Answer. c

146. Which of the following is not the criteria for IgG4 mediated disease?

a. Storiform fibrosis

b. Neutrophil infiltrates

c. Obliterative phlebitis

d. IgG4 levels >135mg/dl

Solution. B: Neutrophil infiltrates


IgG4-related disease (IgG4-RD) is a newly recognized constellation of disorders characterized by
tissue infiltrates dominated by IgG4 antibody-producing plasma cells and lymphocytes

Criteria on biopsy for IgG4 disease

·         Dense Lymphoplasmacytic infiltrate

·         Storiform fibrosis

·         Obliterative phlebitis

Definite diagnosis of IgG4-RD may be made by demonstrating

1.      organ involvement

2.      a serum IgG4 level exceeding 135 mg/dL

3.      greater than 10 IgG4+ plasma cells per high-power field and an


IgG4+:IgG+ plasma cell ratio of at least 40% on histologic tissue sections.

The disease spectrum include Mikulicz syndrome (enlargement and fibrosis of salivary and
lacrimal glands), Riedel thyroiditis, idiopathic retroperitoneal fibrosis, autoimmune pancreatitis,
and inflammatory pseudotumors of the orbit, lungs, and kidneys.

Answer. b

147. Tuberculosis of the spine commonly affects all of the following parts of the vertebra except:

a. Body

b. Lamina

c. Spinous process

d. Pedicle

Solution. c: Spinous process


Facet joints followed by spinous process is least commonly involved in Pott’s spine.

Answer. c

148. In Supracondylar fracture of humerus in children, if the radial pulse is absent, what is next
line of management?

a. Emergency brachial artery exploration

b. Closed reduction of fracture and look of reappearance of pulse

c. Closed reduction, above elbow slab plaster and observe

d. Open reduction and internal fixation of fracture.

Solution. b: Closed reduction of fracture and look of reappearance of pulse.


Closed reduction and flex the elbow till the pulse reappears, and then immobilize the elbow in
that position.

Answer. b
149. All the following are causes of a painful limp except :

a. Slipped capital femoral epiphysis

b. Tuberculosis of the hip

c. Perthes disease

d. Infantile Coxa Vara

Solution. d: Infantile Coxa Vara


Perthes disease causes painless limp initially, but later may become painful, while infantile
(congenital) coxa vara is characteristically painless .

Answer. d

150. Game Keeper’s thumb is:

a. Ulnar collateral ligament injury of MCP joint

b. Radial collateral ligament injury of MCP joint

c. Radial collateral ligament injury of CMC joint

d. Ulnar collateral ligament injury of CMC joint

Solution. a: Ulnar collateral ligament injury of MCP joint.


Injury to thumb metacarpophalangeal joint ulnar collateral ligament.

Answer. a

151. Patient presents with knee problem. He gives history of injury during playing hockey 3
months back. On testing knee was unstable anteriorly in extension but was stable in 90 degree
of flexion. Probably injury involves?

a. ACL Anteromedial bundle

b. ACL Posterolateral bundle

c. PCLAnteromedial bundle

d. PCL Posterolateral bundle

Solution. b: ACL Posterolateral bundle


ACL has two bundles : Antero-medial: tight in flexion and posterolateral (Bulky), which is tight in
extension. (Provides principal resistance to hyperextension).

Answer. b

152. In fasciotomy for compartment syndrome which structures are released?

a. Skin

b. Skin, subcutaneous tissue.

c. Skin, subcutaneous tissue, superficial fascia.


d. Skin, subcutaneous tissue, superficial fascia and deep fascia.

Solution. d: Skin, subcutaneous tissue, superficial fascia and deep fascia


Complete release of fascia is important to decompress the closed compartment of increased
pressure.

Answer. d

153. Toddler fracture involves:

a. Femur

b. Tibia

c. Fibula

d. Talus

Solution. b: Tibia
Toddler fracture- seen in children - minimally displaced spiral/oblique fracture of tibia without
fracture of fibula and is also known as CAST ( Childhood Accidental Spiral Tibial) fracture.

Answer. b

154.

Identify the X-ray and diagnosis attached to it :

a. Aneurysmal bone Cyst

b. Simple Bone Cyst

c. Giant Cell Tumor

d. Chondroblastoma

Solution. c: Giant Cell Tumor


There are four characteristic radiographic features when a giant cell tumour is located in a
long bone:
-- Occurs only with a closed growth plate.
-- Abuts articular surface: 84-99% come within 1 cm of the articular surface .
-- Well defined with non-sclerotic margin.
-- Eccentric.

Answer. c

155. Which of the following is not a correct match ?


a. Steida Fracture

b. Pseudo Jones Fracture

c. Hoffas’ Fracture

d. Bumper’s Fracture

Solution. b: Pseudo Jones Fracture


Answer. b

156. A patient presents with loss of sensation over lateral three and a half fingers. Which of the
following will be seen additionally :

a. Loss of sensation over thenar eminence

b. Loss of sensation over hypothenar eminence

c. Atrophy of adductor pollicis

d. Opponens palsy

Solution. d :Opponens palsy

Answer. d

157. Walking up and downstairs with support usually begins at what age?

a. 15 months

b. 18 months

c. 24 months

d. 30 months

Solution. B: 18 months
Ref: Nelson 20th Ed, WHO-MGRS
Explanation: Stair walking
• Crawls/creeps upstairs: 15 months
• Walking upstairs and downstairs with support: 18 months
• Walking upstairs without support (2 feet at each step): 24 months
• Walking upstairs without support (Alternate feet): 30 months
• Walking downstairs without support (2 feet at each step): 36-42 months
• Walking downstairs without support (Alternate feet): 44-48 months

Answer. b

158. Gas bubbles in portal vein first appear in what stage of necrotising enterocolitis?

a. IIa

b. IIb

c. IIIa

d. IIIb

Solution. B: IIb
Ref: Cloherty’s textbook, Avery
Explanation: Modified Bell’s staging for NEC
Answer. b

159. Which among the following is likely to be the most common heart disease in the child
shown below if he also shows low IQ and hypercalcemia?

a. Pulmonic stenosis

b. Supravalvular aortic stenosis

c. ASD

d. Bicuspid aortic valve

Solution. B: Supravalvular aortic stenosis


This child is showing Elfin facies, with low IQ and has hypercalcemia. Hence the diagnosis is
Williams syndrome. The most common heart disease is supravalvular AS.

Answer. b

160. A preterm neonate developed asphyxia but was resuscitated. MRI revealed periventricular
leukomalacia. What is the likely sequela in this child?

a. Spastic diplegic CP

b. Spastic quadriplegic CP

c. Choreoathetoid CP

d. Atonic CP
Solution. A: Spastic diplegic CP
Ref: Nelson 20th Ed/ 2897
Explanation: The most common neuropathologic finding in children with spastic diplegia is PVL,
which is visualized on MRI in more than 70% of cases. MRI typically shows scarring and
shrinkage in the periventricular white matter with compensatory enlargement of the cerebral
ventricles.

Answer. a

161. Not seen in a 5 year old child with cystic fibrosis?

a. Nasal polyps

b. Pseudomonas pneumonias

c. Diabetes mellitus

d. Hypochloremic alkalosis

Solution. C: Diabetes mellitus


Ref: Nelson 20th Ed/2104
Explanation: In addition to exocrine pancreatic insufficiency, evidence for hyperglycaemia and
glucosuria, including polyuria and weight loss, may appear, especially in the 2nd decade of life.
Diabetes mellitus is rare to absent in the first decade of life. Ketoacidosis usually does not occur,
but eye, kidney, and other vascular complications have been noted in patients living ≥10 yr after
the onset of hyperglycaemia. Excess sweat Cl loss may produce hypochloremic metabolic
alkalosis.

Answer. c

162. Drug of choice for chest-indrawing pneumonia in a one-year old child is?

a. Oral amoxicillin

b. Oral cotrimoxazole

c. IV Ampicillin + Gentamicin

d. IV Ceftriaxone

Solution. A: Oral amoxicillin


Ref: Revised WHO guidelines 2015
Explanation: As per the latest recommendations, children age 2–59 months with chest indrawing
pneumonia should be treated with oral amoxicillin: at least 40mg/kg/dose twice daily
(80mg/kg/day) for five days. Previous guidelines by WHO for the management of chest indrawing
pneumonia in children recommended parenteral antibiotics for at least three days. A study by
Straus et al published in 1998 found that treatment failure rate with oral amoxicillin was
significantly lower than with oral cotrimoxazole (18% and 33% respectively) in children with
chest indrawing pneumonia (p=0.009). It was concluded that although oral cotrimoxazole was
effective in fast breathing pneumonia, it was less effective in treating chest indrawing
pneumonia. In resource-limited settings, oral amoxicillin is equally efficacious as injectable
antibiotics and has lower cost with better compliance.

Answer. a

163. Contraindicated in childhood dysentery?


a. Loperamide

b. Ondansetron

c. Probiotics

d. Ciprofloxacin

Solution. A: Loperamide
Ref: Nelson 20th Ed/1873
Explanation: Antimotility agents (loperamide) are contraindicated in children with dysentery and
probably have no role in the management of acute watery diarrhoea in otherwise healthy
children. Similarly, antiemetic agents, such as the phenothiazines, are of little value and are
associated with potentially serious side effects (lethargy, dystonia, malignant hyperpyrexia).
Ondansetron is safe and a useful adjunct in troublesome vomiting. Probiotics can be given but
are not routinely indicated.
Remember: Ciprofloxacin is the drug of choice in childhood dysentery (WHO-SEAR update,
Nelson 20th Ed).

Answer. a

164. Not seen in infant born to mother with uncontrolled diabetes during pregnancy?

a. Poor neurodevelopmental outcome

b. Shoulder dystocia

c. Severe anaemia

d. Renal vein thrombosis

Solution. C: Severe anaemia


Ref: Nelson 20th Ed/897
Explanation:

Answer. c
165. Drug of choice in childhood steroid-resistant nephrotic syndrome?

a. Levamisole

b. Cyclophosphamide

c. Cyclosporine

d. Rituximab

Solution. C: Cyclosporine
Ref: Nelson 20th Ed, ISPN guidelines, KDIGO guidelines
Explanation: Drugs of choice in childhood nephrotic syndrome
• First episode: Prednisolone
• Any relapse: Prednisolone
• Frequent relapses or Steroid dependent nephrotic syndrome: Cyclophosphamide
• Relapse with steroid toxicity: Cyclophosphamide
• Steroid resistant nephrotic syndrome: Calcineurin inhibitors
(Cyclosporine/Tacrolimus)

Answer. c

166. Not seen in Kwashiorkor?

a. Apathy

b. Hepatomegaly

c. Baggy pants appearance

d. Flaky paint dermatosis

Solution. C: Baggy pants appearance


Ref: OP Ghai 8th Ed, IAP review
Explanation: Baggy pants appearance is a feature of marasmus, not kwashiorkor. Baggy pants
appearance occurs due to severe loss of fat from buttocks and thighs due to catabolism. All
others are seen in a child with Kwashiorkar.

Answer. c

167. The Tympanic reflex is an involuntary muscle contraction that occurs in the middle ear in
response to high-intensity sound, reaction time for the reflex is about:

a. 10-20 ms

b. 20-40 ms

c. 40-160 ms

d. 160-200 ms

Solution. (c) 40-160 ms


• The Tympanic reflex is an involuntary muscle contraction that occurs in the middle ear in
response to high-intensity sound stimuli or when the person starts to vocalize.
• Reaction time for the reflex is 40-160 ms.

Answer. c
168. Spot Diagnosis:

a. Quinke's disease

b. Quinsy

c. Palatal hematoma

d. Ca Tongue

Solution. (a) Quinke's disease


Odema of the uvula (Quinke's disease).
Definitions: Quinke's disease is acute oedema of the uvula.

Peritonsillar Abscess (Quinsy):


Commonest site: Upper pole of tonsils, usually unilateral.

Answer. a

169. A hyperemic edema of the larynx and epiglottis that rapidly leads to respiratory
obstruction in young children is most likely to be caused by

a. Streptococcus

b. M. pneumoniae

c. Neisseria meningitidis

d. H. influenzae

Solution. (a) Streptococcus


Epiglottitis
Pathogen: Streptococcus
Risks: children >1 year old to adult (most common between 2–6 years of age)
SSx: sudden onset (hours) and short course, high fever, dysphagia, drooling, dyspnea, “sniffing
position” (neck flexed and head extended), no cough, normal voice, tender larynx
Dx: clinical history and exam, plain neck films (“thumbprint sign”), serum HIB capsule antigen,
cultures
Complications: septicemia, acute airway obstruction (death)
Management
1. AVOID AGGRAVATING PATIENT (do not examine airway with a tongue blade, draw blood,
perform rectal temperatures, etc)
2. Establish Emergent Airway: intubation performed in the operating room with preparation for
a tracheotomy

3. Endoscopy: examine and culture epiglottis


4. Postoperative Care: monitored bed, parenteral antibiotics and corticosteroids for 7–10 days
(consider ampicillin [20% resistance] with chloramphenicol or cefuroxime), consider extubation
after 2–3 days

Answer. a

170. Otoacoustic emissions arise from?

a. Inner hair cells

b. Outer hair cells

c. Organ or Corti

d. Brain stem

Solution. (b) Outer hair cells


• An otoacoustic emission (OAE) is a low-level sound emitted by the cochlea either
spontaneously or evoked by an auditory stimulus.
• Specifically, OAEs provide information related to the function of the outer hair cells (OHC)

Answer. b

171. The adduction of the vocal cords cannot occur while talking but can occur with a good
cough is a condition associated with

a. Dysphonia

b. Temporary aphonia

c. Functional aphonia

d. Complete aphonia

Solution. (c) Functional aphonia


• Psychogenic aphonia is often seen in patients with underlying psychological problems.
• Laryngeal examination will show usually bowed vocal folds that fail to adduct to the midline
during phonation.
• However, the vocal folds will adduct when the patient is asked to cough. Treatment should
involve consultation and counseling with a speech pathologist and, if necessary, a psychologist.
• In this case, the patient's history and the observed unilateral immobility rules out functional
aphonia.

Answer. c

172. Normal volume of middle ear and mastoid is?

a. 1 cc

b. 6 cc

c. 12 cc

d. 15 cc

Solution. (b) 6 cc
• Middle ear volume (MEV), defined as the continuous volume occupied by the tympanic cavity
and mastoid air cells, has been characterized in the setting of various middle ear pathologies
using.
• Determining MEV size by tympanometry has proved clinically useful in various settings.
• The normal, average volume of the middle ear and mastoid is 6 cc

Answer. b

173. Unilateral blood-stained nasal discharge in a child aged three years, is most probably due
to ?

a. Sinusitis

b. Dental infection

c. Viral rhinitis

d. A neglected foreign body

Solution. (d) A neglected foreign body


• Common nasal FBs include hair beads, toy parts, paper, and food.Nasal FBs are usually
asymptomatic.
• Symptomatic patients may present with unilateral nasal occlusion and fetid, purulent, or
blood-stained nasal discharge.
• They may be complicated by unilateral sinusitis or periorbital cellulitis.
• The FB usually can be seen with anterior rhinoscopy after suctioning purulent secretions.
Radiographs are of limited diagnostic value because most FBs are radiolucent.

Answer. d

174. Appearance of fever with rigor in a person with otitis media should make you suspect ?

a. Cerebellar abscess

b. Extradural abscess

c. Lateral sinus thrombosis

d. Apex petrositis

Solution. (c) Lateral sinus thrombosis


• Although seldom encountered in the practice of modern otology, lateral sinus thrombosis (LST)
and other intracranial complications of otitis media still occur.
• The intracranial complications of otitis media include purulent meningitis, extradural or
peridural abscess, LST, brain abscess and otitic hydrocephalus. Respiratory mucosa, intact
boney walls and protective granulations provide natural defense barriers within the middle ear;
complications occur when these are overcome. The spread of infection through the natural
defenses can occur by osteothrombosis, bone erosion and when present along preformed
pathways.
• Classic symptoms of LST include a "picket fence" fever pattern; chills; progressive anemia
(especially with beta-hemolytic strep); and, symptoms of septic emboli, headache and
papilledema may indicate extension to involve the cavernous sinus. The Toby-Ayer test is
measured by monitoring the CSF pressure during a lumbar puncture. No increase in CSF
pressure during external compression of the internal jugular vein on the affected side, and an
exaggerated response on the patent side, is suggestive of LST.

Answer. c

175. In the internal auditory canal, the vertical crest (Bill’s bar) marks a plane between?

a. Superior and inferior vestibular nerves


b. Facial and cochlear nerves

c. Inferior vestibular and singular nerves

d. Superior vestibular and facial nerves

Solution. (d) Superior vestibular and facial nerves


Reference – Otolaryngology by R.Pasha --358
Sol:
• The lateral wall of the IAC is the lamina cribrosa, and the superior fundus is divided vertically
by Bill’s bar, a triangular strut of bone seen only on axial images.

• Facial nerve transverses in the anterior superior quadrant of the IAC separated by the
falciform crest inferiorly and Bill’s bar posteriorly.
Other quadrants
• Superior vestibular nerve [superior posterior]
• Inferior vestibular nerve [inferior posterior]
• The cochlear nerve [inferior anterior]

Answer. d

176. Benign Juvenile papilloma of the larynx in children

a. Is solitary and sessile

b. Has tendency to develop into papillary carcinoma

c. Is multiple and fungating

d. Is a familial inherited disease

Solution. (c) Is multiple and fungating


Benign Juvenile papilloma of the larynx
• Multiple
• Often involves infants & young children
• Mostly seen on true & false cords & epiglottis
• Presents with hoarseness & stridor
• Known for recurrence after removal
• Disappears spontaneously after puberty
• CO2 laser is preferred
• Interferon therapy may be given (to prevent recurrence)

Answer. c

177. The minimal angle of resolution for a patient whose visual acuity on Snellen’s chart is 6/36

a. 4 min of arc

b. 5 min of arc

c. 6 min of arc

d. 10 min of arc

Solution. c : 6 min of arc.


The minimal angle of resolution is just reverse of visual acuity.

Answer. c
178. Which is false for left trochlear nerve nucleus lesion

a. Leads to right superior oblique palsy

b. It is in midbrain at level of inferior colliculus

c. It will lead to hypertropia in the left eye

d. The patient will have left head tilt as compensation

Solution. C: It will lead to hypertropia in the left eye


It is the only motor cranial nerve to decussate, so it will lead to right superior oblique palsy.
The level of trochlear nerve nucleus is at inferior colliculus (oculomotor nucleus is at level of
superior colliculus)
The patient in right SO palsy will have right hypertropia and compensation to avoid diplopia
will be – chin down, left face turn and left head tilt
Ref – Wills eye Manual, 6th edition. Page 247

Answer. c

179. What is the probable diagnosis of the image?

a. Moderate Non- Proliferative Diabetic Retinopathy

b. Proliferative Diabetic Retinopathy

c. Hypertensive retinopathy

d. Exudative Age related macular degeneration

Solution. B: Proliferative Diabetic Retinopathy


The angiography picture of left eye shows neovascularized vessels, which signifies Proliferative
diabetic retinopathy.

Answer. b

180. Gyrate atrophy is a retinal condition that occurs with deficiency of Ornithine
aminotransferase enzyme. Which of the following will have the most benefit for patient of gyrate
atrophy?

a. Ornithine restricted diet

b. Arginine restricted diet

c. Folic acid and cyanocobalamin

d. Vitamin B Complex tablets


Solution. B: Arginine restricted diet
A missense mutation results in the deficiency of a specific enzyme, ornithine ketoacid
aminotransferase. the presenting symptom of ornithine aminotransferase (OAT) deficiency.
Ophthalmological findings in affected individuals include constricted visual fields, posterior
subcapsular cataracts. Arginine free diet is beneficial.

Answer. b

181. Which drug has been approved by FDA for Acanthamoeba ulcers?

a. Miltefosine (Impavido)

b. Netarsudil (Rhopressa)

c. Latanoprosene (Vyzulta)

d. Voretigene (Luxturna)

Solution. A: Miltefosine (Impavido)


Miltefosine (Impavido) - for treatment of acanthamoeba ulcers (65 mcg/ml); orphan drug.
Luxturna (voretigene)- For the treatment of vision loss due to confirmed biallelic RPE65-
mediated inherited retinal disease, Approved December 2017.
Rhopressa (netarsudil)-For the treatment of glaucoma or ocular hypertension, Approved
December 2017.
Vyzulta (latanoprostenebunod ophthalmic solution)-For the reduction of intraocular pressure in
patients with open-angle glaucoma or ocular hypertension, Approved November 2017.
Zerviate (cetirizine ophthalmic solution 0.24%)- For the treatment of ocular itching
associated with allergic conjunctivitis, Approved May 2017

Answer. a

182. On Hirshberg corneal reflex test as shown in the figure, the right eye shows reflex in the
center of pupil, while the left eye has reflex temporal to pupil.What is the probable diagnosis?

a. Right esotropia

b. Left esotropia

c. Right hypotropia

d. Left hypertropia

Solution. B: Left esotropia


Right eye looking straight, while left inside means that the patient has left esotropia, probably
due to left 6th nerve palsy.

Answer. b

183. Identify the type of cataract shown below.


a. Cuneiform cataract

b. Rosette cataract

c. Morgagnian cataract

d. Snow flake cataract

Solution. C: Morgagnian cataract


Cuneiform cataract- type of cortical cataract
Rosette cataract – blunt trauma
Morgagnian cataract – type of hypermature cataract
Snow flake cataract – diabetes mellitus

Answer. c

184. A pregnant woman has open angles and IOP 26, 34 mm Hg (OD, OS) and CDR 0.6, 0.8.
Which of the following should be done?

a. Abort the fetus

b. Laser iridotomy

c. Start brimonidine

d. Start prostaglandins

Solution. C: Start brimonidine


Only brimonidine (alpha agonist) is safe to be given in pregnancy as being the only class B
antiglaucoma drug. Rest are class C.

Answer. c

185. Which of the following represents lesion at right temporal lobe?

a.
b.

c.

d.

Solution. B:

A lesion in right temporal lobe will produce left homonymous upper quandrantopia due to the
damage of the Meyers loop. This will involve upper nasal field in the right eye and upper
temporal field in the left eye.
A is lesion at junction right optic nerve and chiasma (junctional scotoma).
C is left parietal lobe lesion causinfright homonymous lower quandrantopia.
D is bitemporal hemianopia due to optic chiasma lesion.

Answer. b

186. All are subjective tests in ophthalmology except

a. Maddox rod test

b. Snellen’s vision testing

c. Opto-kineto-nystagmogram

d. Worth 4 dot test

Solution. C: Opto-kineto-nystagmogram
A subjective test is evaluated by asking an opinion from the patient.
Optokinetonystagmogram (OKN) is an objective test and is thought to occur because the eyes
are trying to keep a moving image stationary on the fovea. The reflex is fully developed by 5-6
months, but is present in a crude form in newborn infants. When a subject views a rotating
striped drum, his or her eyes involuntarily follow a stripe with a “slow eye movement” (pursuit),
then return with a fast eye movement (saccade) to fixate on a new stripe. The whole cycle can be
repeated indefinitely as long as the subject views the moving drum.

Answer. c
187. Which of the following is avoided in epilepsy patients

a. Scoline

b. Atracurium

c. Mivacurium

d. Rocuronium

Solution. B:ATRACURIUM
Atracurium is metabolised by Hoffman degradation(1/3) and alkaline ester hydrolysis (2/3). It
produces metabolite laudonosine which can cause convulsions. Atrcurium causes histamine
release, is doc for renal failure,hepatic failure, pt with atypical pseudocholinesterase.

Answer. b

188. Macintosh is associated with

a. Endotracheal tubes

b. LMA

c. Laryngoscope

d. Tracheostomy

Solution. C:LARYNGOSCOPE
It is curved laryngoscope blade, straight one is called miller. Laryngoscopy is done in sniffing
position, ie extension at atlanto-occipital joint and flexion in the neck.

Answer. c

189. Endobronchial intubation can be detected by

a. Auscultation

b. Inability to palpate cuff in suprasternal notch

c. Decreased breathing bag compliance

d. Capnography

Solution. D:CAPNOGRAPHY
Golden test for intubation is capnography, normal values are 35-45 mm Hg, capnography
doesnot rule out endobronchial intubation. Best site for auscultation is left base. The cuff should
lie in suprasternal notch 2 to 2.5 cm below vocal cords.

Answer. d

190. Warfarin should be stopped how many days before surgery

a. 1-2 days

b. 2-3 days
c. 3-5 days

d. >7 days

Solution. C:3-5 DAYS


Warfarin should be stopped 4-5 days before surgery, LMWH 12-14 hrs before Sx, and
unfractioned heparin 6 hrs before Sx. In emergency FFP can be used. Low dose aspirin to be
continued except for closed space Sx. Aspirin if need can be stopped 3-5 days before Sx,
clopidogrel 7 days, and ticlopidine 14 days before Sx.

Answer. c

191. Nitrous oxide was first prepared by

a. Humphry davy

b. Joseph priestly

c. Simpson

d. Carl koller

Solution. B:JOSEPH PRIESTLY


Priestly synthesized oxygen and nitrous oxide,davy showed that nitrous oxide can provide pain
relief or analgesia, simpson used chloroform, koller used cocaine for 1st
time.

Answer. b

192. Miliaria is a disorder of:

a. Sebaceous glands

b. Apocrine glands

c. Holocrine glands

d. Eccrine Glands

Solution. D: Eccrine Glands


Miliaria is the retention of sweat as a result of occlusion of eccrine sweat ducts. It is common in
hot, humid climates. Staphylococcus epidermidis, which produces an extracellular
polysaccharide substance, induces miliaria. This polysaccharide substance may obstruct the
delivery of sweat to the skin surface. The occlusion prevents normal secretion from the sweat
glands, and eventually pressure causes rupture of the sweat gland or duct at different levels.
The escape of sweat into the adjacent tissue produces miliaria. Depending on the level of the
injury to the sweat gland or duct, several different forms are recognized.
- Miliaria crystallina (sudamina)
- Miliaria rubra (prickly heat)
- Miliaria pustulosa
- Miliaria profunda

Answer. d

193. A 24 middle aged female has flaccid bullae in the skin and persistent painful oral erosions.
Histopathology shows intraepidermal acantholytic blister. The most likely diagnosis is
a. Bullous Pemphigoid

b. Erythema multiforme

c. Pemphigus vulgaris

d. Dermatitis herpetiformis

Solution. C: Pemphigus vulgaris

Acantholysis is a typical feature of pemphigus. BP, Erythema multiforme and Dermatitis


herpetiformis don’t have acantholysis. Flaccid blisters also point to pemphigus.

Answer. c

194. Which is a correct combination?

a. Erythema infectiosum- TB

b. Erythema marginatum- Parvovirus

c. Erythema multiforme- Sarcoidosis

d. Erythema chronicum - Lyme's disease

Solution. D: Erythema chronicum - Lyme's disease


Erythema infectiosum- parvovirus B19 infection
Erythema marginatum- Rheumatic fever
Erythema multiforme- Reaction pattern to HSV, Drugs, Mycoplasma
Erythema chronicum- Lyme disease

Answer. d

195. Which of these statements is false for the painful lesions shown in the image below?

a. A form of vasculitis

b. Superficial skin inflammation

c. Commonly ulcerates

d. Associated with Streptococcus

Solution. D: Associated with Streptococcus


The image is of erythema nodosum with red tender nodules on shin. It is a form of septal
panniculitis, not vasculitis. They never ulcerate and are commonly associated with streptococcal
pharyngitis.

Answer. d
196. A 10 years old boy presented with painful boggy swelling of scalp, multiple sinuses with
purulent discharge, easily pluckable hair and enlarged lymph nodes in the occipital region.
Which one of the following would be most helpful for diagnosis?

a. KOH mount

b. Biopsy and culture

c. Bacterial culture

d. Patch test

Solution. A: KOH mount


Child age group would support T. capitis diagnosis. Painful boggy swelling would suggest a
diagnosis of Kerion. KOH smear helps in diagnosis.

Answer. a

197. Causes of cicatricial alopecia include:

a. Lupus erythematosus

b. Telogen effluvium

c. Lichen planus

d. Tinea

Solution. B: Telogen effluvium

Answer. b

198. All are true statements regarding leprosy except:

a. Paucibacillary leprosy means is having 1-5 skin lesions

b. Regular MDT means patient received 2/3rd of month of treatment schedule

c. Lepra reaction occurs usually after initiation of treatment

d. Lepromin test is used for treatment initiation

Solution. D: Lepromin test is used for treatment initiation


Lepromin is an intradermal prognostic test in leprosy and has no role in initiation of MDT.

Answer. d

199. The most common association with irregular pitting of nails and onycholysis is:

a. Violaceous papules

b. Extensor plaques

c. Cicatricial alopecia

d. Exclamation hair
Solution. B: Extensor plaques
The nail changes are of psoriasis. Hence B is the answer.

Answer. b

200. Regarding Lichen planus, all are true except

a. Hypopigmentation in residual disease

b. Lymphocytic infiltration

c. Itchy, polygonal, purple, papule

d. Skin, hair and oral mucosa are commonly involved

Solution. A: Hypopigmentation in residual disease


LP leads to hyperpigmentation, not hypopigmentation.

Answer. a

201. Drug used in scabies are all except:

a. Crotamiton

b. Permethrin

c. Lindane

d. Tacrolimus

Solution. D: Tacrolimus

Answer. d

202. Most common gene mutation in dilated cardiomyopathy?

a. Tafazzin

b. Titin

c. Beta myosin heavy chain

d. NOTCH-2

Solution. b : Titin
Important one liners on Dilated cardiomyopathy
1. DCM is the most common cardiomyopathy
2. 30% cases of DCM are familial
3. Most common pattern of inheritance of familial DCM - Autosomal dominant
4. Most common genetic mutation in DCM - Titin gene
5. Most common cause of DCM - Idiopathic
6. Most common identifiable cause of DCM - Alcohol
7. Most common infective cause of DCM - T.cruzi
8. Most common toxin implicated in DCM- Alcohol
9. Most common drug implicated in DCM - Doxorubicin

Answer. b
203. Drug of choice for IgG4 related disease is?

a. Azathioprine

b. Mycophenolate mofetil

c. Cyclophosphamide

d. Steroids

Solution. d : Steroids
Steroids are the drug of choice for IgG4 related disease. Rituximab can be used as add on agent.
Steroid sparing agents like Azathioprine and mycophenolate mofetil are of doubtful benefit only
in IgG4 related disease.

Answer. d

204. A 19-year-old female who has been in good health and takes no chronic medications,
presents to you for new-onset severe fatigue and left upper abdominal pain. She was treated 2
days ago in the health centre for presumed gonorrhoea with ceftriaxone and azithromycin.
Physical examination showed splenomegaly. Stool is negative for heme. Laboratory examination
is notable for a haemoglobin of 5 g/dL with normal WBC and platelets. Peripheral blood smear
shows evidence of haemolysis and excess of spherocytes. Which of the following tests will most
likely confirm the diagnosis?

a. ADAMTS-13 activity assay

b. Direct antiglobulin test

c. Flow cytometry

d. Hemoglobin electrophoresis

Solution. b : Direct antiglobulin test

Answer. b

205. Perifascicular atrophy is a specific feature of?


a. Polymyositis

b. Dermatomyositis

c. Inclusion body myositis

d. All

Solution. b: Dermatomyositis
PERIFASCICULAR ATROPHY is a highly specific feature of muscle biopsies of patients with
dermatomyositis (specificity > 90%) .

Answer. b

206. Which is used in treatment of pure red cell aplasia?

a. Glucocorticoids

b. Cyclosporine

c. Antithymocyte globulin

d. All

Solution. d: All
All are treatment options for pure red cell aplasia.

Answer. d

207. What percentage of patients with generalised myasthenia gravis have anti AchR antibody?

a. 25%

b. 50%

c. 85%
d. 100%

Solution. c: 85%
IMPORTANT TOPIC - ANTIBODIES IN MYASTHENIA GRAVIS
•Antibodies to acetylcholine receptor antibodies (anti-AChR) is virtually diagnostic of MG, but
negative test does not exclude the disease.
•Anti-AChR antibodies are detected in the serum of ~ 85 % of all myasthenic patients,but only in
about 50 % of patients with weakness confined to the ocular muscles.
•40 % of generalised MG patients who are negative for anti-AChR antibody have antibodies to
MuSK.
•MuSK antibodies are rarely present in anti-AChR antibody positive patients or in patients with
ocular MG.
•Patients with MuSK antibody positive MG responds less well to thymectomy than those with
AChR antibody.
•Patients with MuSK antibody positive MG responds less well to anticholinesterase medications
than those with AChR antibody

Answer. c

208. Most common gene mutation implicated in familial amyloid polyneuropathy?

a. Transthyretin

b. Apolipoprotein A1

c. PARK-1

d. Gelsolin

Solution. a: Transthyretin
Familial amyloid polyneuropathy is caused by mutations in the gene coding for transthyretin,
apolipoprotein A1 or gelsolin. Most commonly it is due to transthyretin
mutation. It is predominantly an axonal neuropathy.

Answer. a

209. Hatchet facies is seen in?

a. Systemic sclerosis

b. Myasthenia gravis

c. Hypothyroidism

d. Myotonic dystrophy

Solution. d: Myotonic dystrophy


The prominent temporalis, masseter and facial muscle atrophy and weakness causes “hatchet
facies” in myotonic dystrophy

Answer. d

210. “Blue man syndrome” is related to the use of which drug?

a. Metoprolol

b. Amiodarone
c. Vancomycin

d. Chloramphenicol

Solution. b: Amiodarone
Long term amiodarone therapy is associated with dermatological side effects. Most common is
photosensitivity. Sometimes, there can be a bluish-slate gray discoloration of the skin ( called
"blue man syndrome"), which is most prominent on the face.

Answer. b

211. Which of the following is a calcium channel defect?

a. Hypokalemic periodic paralysis

b. Hyperkalemic periodic paralysis

c. Paramyotonia congenita

d. Anderson Tawil syndrome

Solution. a: Hypokalemic periodic paralysis


Hypokalemic periodic paralysis – Calcium channel
Hyperkalemic periodic paralysis and paramyotonia congenita – Sodium channel
Anderson Tawil syndrome – Potassium channel

Answer. a

212. Which of the following conditions is not associated with calcium pyrophosphate crystal
deposition disease?

a. Hemochromatosis

b. Hypermagnesemia

c. Gitelman’s syndrome

d. Hypophosphatasia

Solution. b: Hypermagnesemia
Conditions associated with CPPD disease are :
- Primary hyperparathyroidism
- Hemochromatosis
- Hypophosphatasia
- Hypomagnesemia
- Gitelman’s syndrome

Answer. b

213. Which of the following is not a genetic cause of chorea?

a. Ataxia telangiectasia

b. Spinocerebellar ataxia

c. Facioscapulohumeral dystrophy

d. Neuroacanthocytosis
Solution. c: Facioscapulohumeral dystrophy
The genetic causes of chorea are :
- Neuroacanthocytosis
- Ataxia telangiectasia
- Spinocerebellar ataxia
- Dentatorubropallidoluysian atrophy
- Benign hereditary chorea

Answer. c

214. Most common type of Membranoproliferative glomerulonephritis?

a. Type I

b. Type II

c. Type III

d. Type IV

Solution. a:Type I
Most common type of MPGN is type I. Type I is commonly associated with persistent
hepatitis C infections, autoimmune diseases or neoplastic diseases.

Answer. a

215. False statement regarding paroxysmal nocturnal haemoglobinuria ?

a. Congenital condition

b. Intravascular haemolysis

c. Tendency for venous thrombosis


d. Eculizumab is used in treatment

Solution. a: Congenital condition


PNH is an acquired chronic haemolytic anaemia characterized by persistent intravascular
haemolysis with recurrent exacerbations. In addition to haemolysis, there is often pancytopenia
and a distinct tendency to venous thrombosis.

Answer. a

216. The ECG pattern shown below is consistent with :

a. First degree heart block

b. Right bundle branch block

c. P-pulmonale

d. Left bundle branch block

Solution. b: Right bundle branch block


This ECG exhibits classic findings of a right bundle branch block with an rSR’ in V 1  and
broad-based terminal S wave in I, II, and V6.  Left bundle branch block would have a similarly
wide QRS, but with the terminal deflections on the QRS occurring with a leftward and posterior
vector leading to a large R pattern in I and V6 and a QS pattern in V1. Ppulmonale is a term used
to indicate right atrial abnormality or enlargement, seen on ECG as a tall P wave in lead II or V 1
(>2.5 mm). First-degree heart block is seen as a PR interval >200 msec (one large box on a
standard- speed ECG).

Answer. b

217. A 56-year-old construction worker with hypertension and a prior history of tobacco abuse
presents to the emergency department with 30 minutes of acute-onset nausea, dyspnoea and
chest pressure. His initial ECG is shown. All of the following are present in this ECG EXCEPT:

a. Inferior myocardial ischemia

b. Posterior myocardial ischemia

c. Sinus tachycardia

d. Ventricular tachycardia

Solution. d: Ventricular tachycardia


Patient is suffering an acute inferolateral myocardial infarction (MI). Note the ST elevations in
the inferior leads (II, III, aVF) and in V6 (lateral precordial lead). Also, the striking ST
depressions in the anterior precordial leads (V1 –V4) are indicative of posterior ischemia. Here,
the presence of ST depressions in an anterior lead represents the “mirror” of ST elevations in a
posterior location. One can visualize the ECG pattern in the anterior leads upside down to see
the reciprocal nature of the ST depressions. Although on initial glance, the QRS pattern appears
wide in the anterior leads suggesting ventricular tachycardia, on further examination, there are
P waves present that are associated with each QRS (seen most clearly in leads II and V 1); thus,
the rhythm is sinus tachycardia.

Answer. d

218. Preferred option for treatment of alcoholic hepatitis with discriminant function above 32?

a. Steroid

b. Azathioprine

c. Pentoxifylline

d. Cyclophosphamide

Solution. a: Steroid

Answer. a

219. Which of the following is a small molecule inhibitor that acts in rheumatoid arthritis by
inhibiting JAK1 and JAK3 ?

a. Tofacitinib

b. Tocilizumab

c. Rituximab
d. Abetacept

Solution. a: Tofacitinib
Tofacitinib is a small molecule inhibitor that primarily inhibits JAK1 and JAK3 which mediate
signalling of the receptors for the common cytokines that promote inflammation in Rheumatoid
arthritis.

Answer. a

220. Which of the following diseases produces this characteristic skin change ?

a. SLE

b. Systemic sclerosis

c. Sjogren syndrome

d. Sarcoidosis

Solution. b: Systemic sclerosis


Salt and pepper skin is a feature of systemic sclerosis. The depigmentation spares the
peri-follicular areas.

Answer. b

221. Not a feature of Carney complex?

a. Pituitary hyperplasia

b. Atrial myxomas

c. Hyperparathyroidism

d. Schwannomas

Solution. c: Hyperparathyroidism
Carney complex is an autosomal dominant disorder. The gene mutation implicated is
PRKAR1A.
Features are :
- Cushing’s syndrome
- Pituitary hyperplasia and adenomas
- Atrial myxomas
- Schwannomas
- Adrenal hyperplasia
- Lentigines

Answer. c
222. Hunt-Hess scale and World Federation of Neurologic Society Scale are used for?

a. ST elevation MI

b. Non-ST elevation MI

c. Extradural hemorrhage

d. Subarachnoid haemorrhage

Solution. d: Subarachnoid haemorrhage

Answer. d

223. Which of the following is not a feature of extramedullary tumour?

a. Early corticospinal signs

b. Root pain

c. Abnormal CSF findings

d. Sacral sparing

Solution. d: Sacral sparing


Answer. d

224. Most common mutation found in MODY is :

a. HNF 4 alpha

b. Glucokinase

c. HNF 1 alpha

d. HNF 1 beta

Solution. c: HNF 1 alpha


Answer. c

225. The following device is used in?

a. Removal of clot from cerebral circulation

b. Removal of clot from coronary circulation

c. Ventricular pacing

d. Closure of ASD

Solution. d: Closure of ASD


This is the Amplatzer Septal Occluder device used in closure of ASD.

Answer. d

226. The following murmur pattern is seen in?


a. Aortic stenosis

b. Ventricular septal defect

c. Aortic regurgitation

d. Mitral regurgitation

Solution. a: Aortic stenosis


Ejection systolic/Midsystolic murmur:
- Crescendo-decrescendo configuration
- Begins after S1 and ends before S2
- Seen in Aortic stenosis, Pulmonic stenosis,HOCM, flow murmurs

Answer. a

227. Which of the following neurologic disease can have the peripheral smear finding?

a. Alzheimer’s disease

b. Parkinson’s disease

c. Steinert’s disease

d. Abetalipoproteinemia

Solution. d: Abetalipoproteinemia
The peripheral smear shows acanthocytosis.
Three neurological syndromes are associated with acanthocytes in peripheral smear
---> Abetalipoproteinemia
---> Neuroacanthocytosis
---> McLeod syndrome

Answer. d

228. The following is the umbilical artery Doppler of a woman with preeclampsia at 35 weeks of
POG, what is the management?

a. Follow up after 1 week

b. Dexamethasone

c. Termination of pregnancy
d. Continue pregnancy upto 37 weeks

Solution. C Termination of pregnancy


This is a graph of absent end diastolic flow in the umbilical artery and at POG more than 34
weeks. It is an indication for termination of pregnancy. At less than 34 weeks, the pregnancy can
be continued with further monitoring.

Answer. c

229. All the following are correct about progesterone only pills except

a. Consistently inhibit ovulation

b. Make endometrium thin

c. Have to be taken at same time

d. Safe for lactating mothers

Solution. A Consistently inhibit ovulation


Progesterone only pills or minipills have very low amount of progesterone and they do not
inhibit ovulation consistently and approximately half of the women still ovulate. The main
mechanism of action of POP is to reduce the volume of cervical mucus and increase its viscosity
which prevents sperms from passing through the cervical canal. The progesterone only pills also
make the endometrium thin.

Answer. a

230. The following are the contents of broad ligament except

a. Fallopian tube

b. Uterine and ovarian artery

c. Ovarian ligament

d. Internal iliac artery

Solution. D Internal iliac artery


The contents of broad ligament are
1) Fallopian tubes
2) Ovarian vessels
3) Uterine vessels
4) Round ligament
5) Ovarian ligament
6) Epoophoron with its duct
7) Paroophoron
8) Connective tissue(parametrium) with nerves and lymphatics

Answer. d

231. A 44 year old woman has undergone hysterectomy for menorrhagia and dysmenorrhoea
and the following is her uterus specimen. What is the likely pathology?
a. Fibroid uterus

b. Bicornuate uterus

c. Adenomyosis

d. Endometrial cancer

Solution. C Adenomyosis
This is a globular uterus with symmetrical growth, this combined with symptoms and age is
characteristic to adenomyosis. The treatment of choice for adenomyosis is hysterectomy.

Answer. c

232. All the following are true about cervical incompetence except

a. Recurrent first trimester losses

b. Internal os diameter more than 2 cm

c. Painless cervical dilatation

d. Cerclage recommended at 12-14 weeks

Solution. A Recurrent first trimester losses


Cervical incompetence characteristically causes second trimester recurrent abortions and not
first trimester. The diagnosis can be history based or ultrasound based. The ultrasound should
be done in second trimester and we look for cervical length which should be less than 25mm and
internal os diameter more than 2 cm. Cervical incompetence causes painless cervical dilatation.

Answer. a

233. All are risk factors for ectopic pregnancy except

a. Previous ectopic

b. Previous IUD use

c. History of tubal surgery

d. History of PID

Solution. B Previous IUD use


Previous history of IUD use is not a risk factor for ectopic, it is current use of IUD which
increases the relative risk of ectopic pregnancy. Highest risk of ectopic is seen with previous
ectopic which increases risk by 15% followed by history of tubal surgery. Most common risk
factor for ectopic is PID.

Answer. b
234. A woman with preeclampsia after vaginal delivery presents with PPH and was given
oxytocin but continues to bleed what is the next step in the management of this patient

a. Give injection methergin

b. Give sublingual misoprostol

c. Give injection carboprost

d. Do balloon tamponade

Solution. C Give injection carboprost


The patient is bleeding despite the initial treatment with oxytocin and she needs to be given
other uterotonics since the woman is hypertensive. Misoprostol is contraindicated in her and
hence the preferred drug for her is carboprost. 0.25mg is given I/M and the drug can be
repeated every 15mins, misoprostol is indicated when other injectable uterotonics are not
available.

Answer. c

235. All the following are surgeries for vault prolapse except

a. Latzko

b. Sacrocolpopexy

c. Sacrospinous fixation

d. Le Forts repair

Solution. A Latzko
Latzko is a surgery for repair of VVF and not for vault prolapse. Leforts repair or vaginal
obliteration can be done for vault prolapse when longer surgeries cannot be done .The various
surgeries done for vault prolapse are sacrospinous ligament suspension, modified Mc Calls
culdoplasty, uterosacral ligament suspension and sacrocolpopexy and iliococcygeus suspension.

Answer. a

236. Which of the following is not associated with increased risk for abruption ?

a. Smoking

b. Premature Rupture of Membranes

c. Preeclampsia

d. Primigravida

Solution. D Primigravida
Increasing parity is a risk factor for abruption and not primigravida. The highest risk of
abruption is with history of previous abruption as the risk of recurrence is 10-15 fold higher.
Hypertensive woman have 5 fold higher risk. Smoking increases the risk by 2.5 fold. Abdominal
trauma, polyhydramnios and PROM are other important risk factors

Answer. d

237. All are indications for diagnostic amniocentesis except


a. TTTS

b. Karyotyping

c. Fetal lung maturation

d. Neural tube defects

Solution. A TTTS
Diagnosis of TTTS is based on ultrasound and not on amniocentesis, amniocentesis may be done
in TTTS to relieve symptoms. Amniocentesis is the safest method for obtaining sample for
karyotyping and is done between 16-18 weeks most commonly. For neural tube defects, we look
for amniotic fluid acetylcholinesterase and AFP. We assess the L/S ratio in the amniotic fluid for
fetal lung maturation.

Answer. a

238. All the following drugs can be given for acute hypertension in pregnancy except

a. Labetalol

b. Intravenous NTG

c. Methyldopa

d. Hydralazine

Solution. C Methyldopa
a) The drugs to be used for acute hypertension in pregnancy are intravenous labetalol as first
line therapy as it is effective, has a rapid onset and good safety profile. The other drugs to be
used are hydralazine, nifedipine and nicardipine, NTG is an option for treatment of hypertension
with pulmonary edema. Sodium nitroprusside is administered as last resort. Methyldopa is the
safest antihypertensive but it has a delay in the onset of action and is also a mild hypertensive
and hence is not used in the management of acute hypertension.

Answer. c

239. A woman with PCOS is high risk for all the following except

a. Endometrial cancer

b. Osteoporosis

c. CAD

d. Type 2 diabetes mellitus

Solution. B Osteoporosis
Women with PCOS have either normal estrogen level or higher estrogen level and hence they
are not an increased risk of osteoporosis.
Long term risks in women PCOS are :
1) Type 2 DM
2) Hypertension
3) CAD
4) Endometrial cancer
5) Ovarian cancer
6) Non alcoholic steatohepatitis
7) Psychiatric disturbances
8) Sleep apnea
9) Metabolic syndrome
10) Dyslipidemia

Answer. b

240. Which of the following is the first maneuver in the management of shoulder dystocia ?

a. Zavanelli

b. Delivery of posterior arm

c. Suprapubic pressure

d. McRoberts

Solution. D McRoberts
Mc Roberts alone is the first maneuver followed by Mc roberts with suprapubic pressure.
The other maneuvers are removal of posterior arm or shoulder(jaquemiers maneuver).
Woods corkscrew and rubins are rotational maneuvers.
Gaskin all fours and clavicle fracture .
The last resort is zavanelli method which needs repositioning the head in the pelvis followed by
CS.

Answer. d

241. Not true about fetal macrosomia is

a. EBW >=4kg is the criteria

b. Associated with beckwith-wiedman syndrome

c. Excessive maternal weight gain is a risk factor

d. Abdominal circumference more than 30cm

Solution. D Abdominal circumference more than 30cm

For Fetal macrosomia on ultrasound we measure abdominal circumference. AC more than 35


alone is predictive of fetal macrosomia.
EBW more than 4kg is the Indian guideline for defining macrosomia and the best estimate of
fetal weight is done by hadlocks formula which uses BPD, FL and AC together.
Maternal diabetes is an important risk factor and others are maternal obesity, multiparity,
advanced maternal age , post term pregnancy, male infant, excessive gestational weight gain
and ethnicity.
Macrosomia is defined as EBW above 90th percentile or >=4kg for india .

Answer. d

242. Which statement is not true regarding maternal physiology in pregnancy ?

a. Exertional dyspnoea

b. Neutrophilic leucocytosis

c. Fasting hyperglycaemia

d. Proteinuria up to 300mg/day
Solution. C Fasting hyperglycaemia
Pregnancy is a state of insulin resistance and hence is characterised by fasting hypoglycaemia
and postprandial hyperglycaemia. There is exertional dyspnoea due to hemodilution and
proteinuria upto 300mg/ day can be seen as physiological changes.

Answer. c

243. 16 year old girl presents with primary amenorrhoea with breast development
corresponding to tanner 2. On ultrasound, uterus is absent and pubic and axillary hair are
sparse. Which of the following is the likely diagnosis?

a. Mullerian agenesis

b. Androgen insensitivity syndrome

c. Turners syndrome

d. Imperforate hymen

Solution. B Androgen insensitivity syndrome


This is a case of primary amenorrhoea with thelarche. Uterus is absent hence it can be AIS or
mullerian agenesis but pubic and axillary hair are sparse hence the diagnosis is AIS, as in
mullerian agenesis, the pubic and axillary hair development is normal. In both turners and
imperforate hymen , the uterus is present.

Answer. b

244. The following are true about the type of breech shown except

a. The risk of cord prolapse is 5%

b. More common in primigravida

c. Can be confused with face presentation

d. Can be delivered vaginally

Solution. A. The risk of cord prolapse is 5%


This is a frank breech and the risk of cord prolapse is least in frank breech, which is 0.5% while
the risk is highest in footling breech and it is 5%. Frank breech is more common in primigravida.
Frank breech is also the most common type of breech at term.

Answer. a
245. Most reliable indicator of dichorionic twin pregnancy on ultrasound is?

a. Two separate placenta

b. Intertwin membrane

c. Lambda sign

d. Number of layers

Solution. A Two separate placenta


Two separate placenta on ultrasound is the most reliable indicator of dichorionic twins.
The other indicators are intertwin membrane with twin peak sign and 4 layers.
The ideal time to do ultrasound for chorionicity is after 7 weeks in the first trimester.

Answer. a

246. All the following are used for screening for cervical cancer except

a. PAP smear

b. Colposcopy

c. VIA

d. HPV testing

Solution. B Colposcopy
Colposcopy is not a done for screening it is done for follow up of an abnormal PAP smear.
HPV DNA testing, VIA and PAP smear are used for screening of cervical cancer. HPV test has
higher
sensitivity while PAP has higher specificity
A cotest is when PAP and HPV are performed together while reflex test is when HPV is used as a
follow up test .
VIA can be used either alone or with HPV for cervical cancer screening in resource limited
countries.

Answer. b

247. which of the following is not protective for endometrial cancer

a. Smoking

b. OCP

c. Breast feeding

d. Tubal ligation

Solution. D Tubal ligation


Tubal ligation is protective for ovarian cancer but not for endometrial cancer. For endometrial
cancer, tubal ligation may be associated with lower stage prognosis at the time of diagnosis.
OCP reduces the risk of endometrial cancer by 50-60%. Smoking is also protective for
endometrial cancer. Physical activity and green tea have also been shown to reduce risk of
endometrial cancer.

Answer. d
248. Which of the following is not a germ cell tumour of the ovary?

a. Dysgerminoma

b. Endodermal sinus tumour

c. Fibroma

d. Embryonal cancer

Solution. C Fibroma

FIBROMA is not a germ cell tumour of ovary. It is sex cord stromal tumour. It is a solid ovarian
tumour and is also associated with meigs syndrome.

Dysgerminoma has the best prognosis among germ cell malignancies, while endodermal sinus
tumour has the worst prognosis.

Germ cell tumours are unilateral and most commonly occur in younger girls between 10-30
years of age.

Answer. c

249. Which of the following statements is not true about the reproductive hormones ?

a. LH surge precedes ovulation

b. Progesterone peaks 8 days after ovulation

c. Estrogen peak precedes LH surge

d. FSH peaks in the luteal phase

Solution. D FSH peaks in the luteal phase


FSH levels are least in the luteal phase and highest in follicular phase, estrogen peak is seen 36
hrs before ovulation and causes LH surge. Peak progesterone levels are seen 8 days after
ovulation. The gonadotropin levels are lowest in the late luteal phase while the leptin levels are
highest in this phase.

Answer. d

250. A 36 year old woman presents with amenorrhoea for 6 months and infertility. Her lab
evaluation shows FSH levels 45IU. All of the following statements are correct for her
management except

a. She needs HRT

b. She needs surrogate mother

c. A karyotype should be performed

d. She has high risk for fractures

Solution. B She needs surrogate mother


This is a case of premature ovarian failure as suggested by her age and FSH levels more than 40
IU So for her fertility she needs donor ovum and not a surrogate mother. She also needs HRT
which will be given up to the natural age of menopause. Because of estrogen deficiency she is
likely to suffer from osteoporosis and is at increased risk of fractures. All women with premature
ovarian failure must undergo karyotyping as a part of their assessment.
Answer. b

251. The following is the HSG film of a 32 year old woman who presents with primary infertility. What is the treatment
of choice for her?

a. Hysteroscopy

b. IUI

c. IVF

d. GIFT

Solution. C IVF

This is an HSG image of bilateral hydrosalpinx and the treatment of choice for this is IVF. IUI
and GIFT cannot be done when there is tubal factor infertility as they need patent fallopian
tubes.
Hysteroscopy is done as management of proximal tubal or cornual block but not for distal block.

Answer. c

252. All the following are long term complications of PID except

a. Infertility

b. Ectopic pregnancy

c. Chronic pelvic pain

d. Cervical cancer

Solution. D Cervical cancer


PID is not a risk factor for cervical cancer although it has shown to increase the risk of ovarian
cancer. The long term complications include recurrent PID, hydrosalpinx , chronic pelvic pain,
infertility, ectopic pregnancy.

Answer. d

253. The following statements are true about Pre Implantation Genetic Testing except

a. Genetic material is obtained from day3 blastocyst trophoectoderm

b. Can be done for women with RPL

c. Requires IVF
d. Useful for stem cell therapy

Solution. A Genetic material is obtained from day3 blastocyst trophoectoderm


The genetic material for Pre Implantation Genetic Testing (PIGT) is taken from day 5 blastocyst
trophoectoderm. The DNA is then amplified and then the genetic testing is done. PIGD cannot be
done with natural methods of conception and needs IVF. The current indications of PIGD are
1) To look for single gene disorders like sickle cell, cystic fibrosis, haemophilia.
2) Couples who want a child with compatible HLA for stem cell therapy for a sibling.
3) Couples who want to avoid a sex linked disorder in the offspring.
4) Couples with recurrent pregnancy loss with balanced translocation.

Answer. a

254. Regarding red degeneration and fibroids, incorrect is ?

a. Presents as acute abdomen

b. Treatment with antibiotics

c. Most common in second trimester

d. Rapid growth is the cause

Solution. B Treatment with antibiotics


RED DEGENRATION presents as acute abdomen in the pregnant woman typically in the second
trimester in a woman who is a known case of fibroid uterus. Rapid growth causes ischaemia and
necrosis in the fibroid. Red degeneration is managed conservatively, it is an aseptic condition
and does not need antibiotics. No operative intervention is required for red degeneration.

Answer. b

255. Regarding acute fatty liver of pregnancy, not true is ?

a. Typically presents in third trimester

b. Management is delivery

c. It is a clinical diagnosis

d. Enlarged liver

Solution. D Enlarged liver


Acute fatty liver of pregnancy presents in the third trimester and the presentation can be similar
to HELLP but AFLP is a more severe form of liver injury. It is a clinical diagnosis and causes
acute liver failure and has a high mortality rate. Although there is fatty infiltration of the liver, it
is shrunken in size and is not enlarged. It is a rare disorder with incidence of 1 in 7000 to 1 in
20000 deliveries The risk factors are previous AFL, multiple gestation and male sex of the fetus.
The initial management of AFLP is prompt delivery irrespective of gestational age. All women
with AFLP and their children must undergo molecular testing for LCHAD, for most common
mutation G1528C.

Answer. d

256. Incorrect about oligohydramnios is

a. AFI <8
b. Associated with Bilateral renal agenesis

c. Seen in potters syndrome

d. Seen with streeter dysplasia

Solution. A AFI <8


OLIGOHYDRAMNIOS is defined as AFI less than 5. Decreased urine production by the fetus
leads to oligohydramnios and hence conditions that are associated with reduced uteroplacental
circulation will also be associated with oligohydramnios. Post term pregnancy and chromosomal
abnormalities are also associated with oligohydramnios. There is no long term treatment for
oligohydramnios but maternal treatment with intravenous isotonic solutions, oral hydration or
amnio infusion can lead to short term improvement.

Answer. a

257. What is the next step in the evaluation of a 32 year old woman who presents with
progressive dysmenorrhoea and bilateral adnexal masses?

a. Diagnostic laparoscopy

b. TVS

c. MRI

d. CA125

Solution. B TVS
The IOC for adnexal mass is TVS. This is most likely a case of endometriosis with bilateral
chocolate cyst. TVS will further confirm the presence of adnexal mass and its nature, which will
guide the further treatment planning of the patient. CA 125 is done in premenopausal women if
ultrasound appearance of mass raises sufficient suspicion of malignancy.

Answer. b

258. Most common personality disorder?

a. Borderline Personality Disorder

b. Histrionic Personality Disorder

c. Obsessive Compulsive Personality Disorder

d. Narcissistic personality disorder

Solution. C. OCPD (Ref : Kaplan)


Explanation:
"DSM-5 reports an estimated prevalence ranging from 2 to 8 percent for OCPD."
"Borderline personality disorder is thought to be present in about 1 to 2 percent of the
population"
"Data from general population studies suggest a prevalence of histrionic personality
disorder of about 1 to 3 percent."
"According to DSM-5, estimates of the prevalence of narcissistic personality disorder range from
less than 1 to 6 percent"

Answer. c

259. A patient was asked where she lived and she replied: ‘Birmingham, Kingstanding; see the
king he’s standing, king, king, sing, sing, bird on the wing, wing, wing on the bird, bird, turd,
turd.’ This is an example of:

a. Derailment

b. Neologism

c. Flight of ideas

d. Perseveration

Solution. C. Flight of ideas (Ref : Fish's Psychopath)


"Flight of ideas In flight of ideas thoughts follow each other rapidly; there is no general direction
of thinking; and the connections between successive thoughts appear to be due to chance
factors which, however, can usually be understood. The patient’s speech is easily diverted to
external stimuli and by internal superficial associations. The progress of thought can be
compared to a game of dominoes in which one half of the first piece played determines one half
of the next piece to be played. The absence of a determining tendency to thinking allows the
associations of the train of thought to be determined by chance relationships, verbal
associations of all kinds (such as assonance, alliteration and so on), clang  associations,
proverbs, maxims and clichés. The chance linkage of thoughts in flight of ideas is demonstrated
by the fact that one could completely reverse the sequence of the record of a flight of ideas, and
the progression of thought would be understood just as well. An example of flight of ideas comes
from a manic patient who was asked where she lived and she replied: ‘Birmingham,
Kingstanding; see the king he’s standing, king, king, sing, sing, bird on the wing, wing, wing on
the bird, bird, turd, turd.’

Answer. c

260. Which of the following is not true about "Othello syndrome"?

a. It is more common in women than men.

b. Seen with cocaine abuse

c. May be a symptom of schizophrenia

d. It is delusion of morbid jealousy

Solution. A. It is more common in women (Ref : Fish Psychopath)


"Othello syndrome, also called morbid jealousy, this is a delusional belief that one’s
spouse/partner is being unfaithful. Occurring in men more than women, it may be present on its
own or as a symptom of schizophrenia, alcohol abuse or cocaine abuse. It is highly dangerous
and may lead to stalking, searching or sometimes violence."

Answer. a

261. Somnambulism is most commonly seen in :

a. Children

b. Young adults

c. Middle aged

d. Elderly

Solution. A Children (Ref : Kaplan)


"Sleepwalking is very common in children and has peak prevalence between ages 4 and 8 years.
After adolescence, it usually disappears spontaneously."

Answer. a

262. What is the most common chief complaint of all men treated for sexual disorders?

a. Erectile dysfunction

b. Premature ejaculation

c. Hypoactive sexual desire

d. Delayed ejaculation

Solution. A: Erectile dysfunction


. (Ref : Kaplan)
"Erectile disorder is the chief complaint of more than 50 percent of all men treated for sexual
disorders.
Premature ejaculation is the chief complaint of about 35 to 40 percent of men treated for sexual
disorders.
The reported prevalence of low desire is greatest at the younger and older ends of the age
spectrum, with only 2 percent of men ages 16 to 44 aected by this disorder. A reported 6 percent
of men ages 18 to 24, and 40 percent of men ages 66 to 74, have problems with sexual desire."

Answer. a

263. The major neurotransmitter hypothesized to be involved in delirium is:

a. Serotonin

b. Norepinephrine

c. Acetylcholine

d. Dopamine

Solution. C. Acetylcholine (Ref : Kaplan)


"The major neurotransmitter hypothesized to be involved in delirium is acetylcholine."

Answer. c

264. The head circumference growth decelerates in which of the childhood onset psychiatric
disorder?

a. Rett's syndrome

b. Heller's syndrome

c. Infantile Autism

d. Asperger's syndrome

Solution. A. Rett's syndrome (Ref : Kaplan)


"Rett's syndrome:
During the first 5 months after birth, infants have age-appropriate motor skills, normal head
circumference, and normal growth. Social interactions show the expected reciprocal quality. At 6
months to 2 years of age, however, these children develop progressive encephalopathy with a
number of characteristic features. The signs often include the loss of purposeful hand
movements, which are replaced by stereotypic motions, such as handwringing; the loss of
previously acquired speech; psychomotor retardation; and ataxia. Other stereotypical hand
movements may occur, such as licking or biting the fingers and tapping or slapping. The head
circumference growth decelerates and produces microcephaly. All language skills are lost, and
both receptive and expressive communicative and social skills seem to plateau at developmental
levels between 6 months and 1 year. Poor muscle coordination and an apraxic gait with an
unsteady and stiff quality develop. Associated features include seizures in up to 75 percent of
aected children and disorganized EEG ndings with some epileptiform discharges in almost all
young children with Rett syndrome, even in the absence of clinical seizures. An additional
associated feature is irregular respiration, with episodes of hyperventilation, apnea, and breath
holding."

Answer. a

265. Which is true about BI-RADS ?

a. There are 5 categories BI-RADS I to V.

b. Likelihood of cancer is <2 % in BIRADS III

c. Biopsy proven breast cancer Is BIRADS V

d. BI-RADS VI category has 95-99% chance of breast carcinoma

Solution. B: Likelihood of cancer is <2 % in BIRADS III


Classification:
The latest version "BI-RADS Atlas 2013" classifies lesions into seven categories:
BI-RADS 0:
incomplete, further imaging or information is required
BI-RADS I: negative - absolutely normal
BI-RADS II: benign findings- likelihood of cancer essentially 0%
BI-RADS III: probably benign, likelihood of cancer essentially 0-2 %, short interval followup
suggested
BI-RADS IV: suspicious abnormality, likelihood of cancer between 2-95 %, tissue diagnosis
advised, further classified into 4a,4b and 4c
BI-RADS V: there is a mammographic appearance which is highly suggestive of malignancy,
likelihood of cancer equal to or more than 95%, tissue diagnosis advised
BI-RADS VI: known biopsy proven malignancy

Answer. b

266. Total Skin electron therapy is the treatment of

a. Basal cell ca

b. Squamous cell ca

c. Mycosis fungoides

d. for bone marrow transplant

Solution. C : Mycosis fungoides


Ref: Perez and Brady’s principles and practice of Radiation Oncology,6 th edition
Total-skin electron irradiation is a modality designed for management of diseases that require
irradiation of the entire skin surface or a significant portion of it. The technique is used most
frequently for treatment of mycosis fungoides. Basal cell ca and squamous cell skin cancers are
treated with electrons,usually
focal(localized) and total skin irradiation
Total body irradiation(TBI)- usually is made use of in the setting of bone marrow transplant
.However photon beams are used and not total skin irradiation.
(photon beams ,with higher energy,can penetrate deeper and irradiate bone marrow where as
electros can be used for irradiation of superficial structures only.

Answer. c

267. Proton Beams have been widely used with clinical advantage in the management of

a. Skull base tumours

b. Paediatric tumours

c. Craniospinal irradiation

d. All of the above

Solution. D : All of the above


Ref: Perez and Brady’s principles and practice of Radiation Oncology,6 th edition
Protons beams – bragg peak – minimal exit dose after target.less irradiation of normal tissues
compared to other types of radiation, thus being the preferred modality for irradiation of
1.Skull base tumours
2.paediatric malignancies
3.cranio spinal irradiation
4. craniophrayngioma
5.para nasal sinus tumours

Answer. d

268. Iridium-192 is characterized by the following EXCEPT:

a. It is a widely used radionuclide for brachytherapy procedures.

b. It has a small source size.

c. It is used for permanent implants.

d. The lower photon energy makes radiation protection easier than radium or cesium.

Solution. C : It is used for permanent implants.


Physics for clinical oncology – Amen Sibtain,Andrewmorgan and Niall Macdougall
Iridium-192 is a used radionuclide in part because of its convenience, due to its small size, its
low photon energy simplifying radiation protection and its ability to be used in remote
afterloaders.
It is used for temporary implants and not used for permanent implants

Answer. c

269. What is the name of structure marked by tip of arrow ?


a. Left main coronary

b. Right coronary

c. Left azygos vein

d. IVC

Solution. A: Left main coronary


The left main coronary artery (LMCA) or left coronary artery (LCA) is one of the two main
arteries that supply the heart with oxygenated blood.
It is a branch of the ascending aorta, with its normal origin in the left aortic sinus, just superior
to the aortic valve
LMCA runs for 5-10 mm as it passes to the left and posterior to the pulmonary trunk before
bifurcating into left anterior descending artery (LAD) and left circumflex artery (LCx)

Answer. a

270. What is the name of the structure marked by tip of the arrow ?

a. Aorta

b. IVC

c. Hemiazygous vein

d. Small bowel loop

Solution. B: IVC
The IVC is formed by the confluence of the two common iliac veins at the L5 vertebral level.
The IVC has a retroperitoneal course within the abdominal cavity. It runs along the right side of
the vertebral column with the aorta lying laterally on the left.
IVC should be distinguished from dilated azygous vein by tracing superiorly. Normal IVC
opacification must be noted to rule out IVC thrombosis.

Answer. b
271. On DEXA, T-score between -1 to -2.5 indicates

a. Osteoporosis

b. Osteopenia

c. Osteopetrosis

d. Normal

Solution. b) Osteopenia
T score <-2.5 is osteoporosis.
Between -1 to -2.5 is osteopenia
DEXA is IOC for bone mineral density assessment.

Answer. b

272. Characteristic radiological finding of leiomyoma of oesophagus is?

a. Shouldering

b. Apple core appearance

c. Crescentic appearance

d. Rat tail appearance

Solution. c) Crescentic appearance


See the characteristic image in barium swallow showing a smooth, crescentic appearance, rest
of the appearance shouldering, apple core and rat tail are carcinoma oesophagus.
Imaging findings:
Smooth, sharply-marginated mass.
Well-defined, intramural (wall) mass lesion that typically intersects wall at 90 degree angle
when viewed in profile.
As a wall lesion, it may narrow the lumen in one plane and widen it in the orthogonal view (a
plane at 90 degrees to the original).

Answer. c

273. What is the name of the structure marked by tip of the arrow ?
a. Thalamus

b. Putamen

c. Head of caudate

d. Globus pallidus

Solution. A: Thalamus
The thalamus is composed of two symmetrical egg-shaped masses which are usually connected
at the midline by a band of grey matter, the inter-thalamic adhesion. The anterior pole narrows
to form the posterior boundary of the interventricular foramen. Thalamus hyperintensity has
many causes like viral infection, venous infarcts.

Answer. a

274. What is the name of the structure marked by tip of the arrow ?

a. Spleen

b. Left Adrenal

c. Stomach

d. Pancreas

Solution. B: Left Adrenal


The adrenal glands lie superior and anteromedial to the kidneys. Each gland is enclosed in the
perirenal fascia and each has a body and two limbs: a medial limb and a lateral limb. Adrenal
adenoma and adrenal metastasis are two common lesions of adrenal gland.

Answer. b

275. A 25 year old male presented with persistent back ache. Symptoms are aggravated by
hyperextension. Coronal section of CT and IVP are given below. Which of the following is a false
statement about this condition?
a. This condition is more common in males

b. Isthmus lies against L4

c. IVP gives flower vase sign

d. Prophylactic Isthmectomy is beneficial

Solution. (d) Prophylactic Isthmectomy is beneficial

Answer. d

276. A young patient presented with acute ureteric colic. USG showed a 4 mm stone at right
uretero-vesical junction without any hydronephrosis. What should be best advice for this
patient?

a. Pain relief and observation

b. Immediate Ureteroscopy and stone removal

c. ESWL

d. open surgery

Solution. (a) Pain relief and observation


• Possibility of spontaneous extrusion for a stone < 5mm in size is > 90%.

Answer. a

277. Choose the most inappropriate statement about testicular treatment.

a. 95% of testicular tumors are germ cell tumor

b. Seminoma is very radiosensitive

c. Extragonadal germ cell tumor has better prognosis and twice the survival as compared to
germ cell tumor.
d. Most common urological tumor of young age

Solution. (c) Extragonadal germ cell tumor has better prognosis and twice the survival as
compared to germ cell tumour.

Answer. c

278. A 27-year-old pregnant women came with history of thyroid swelling. FNAC report was
suggestive of papillary carcinoma thyroid and the nodule was “cold” on scan. Which of the
following will not be a treatment option for her?

a. Radioiodine ablation of thyroid

b. Total thyroidectomy

c. Right hemithyroidectomy

d. Total thyroidectomy plus lymph node dissection

Solution. (a) Radioiodine ablation of thyroid


RADIO-IODINE has a limited role as primary management of the carcinomas in thyroid and
also
Radioiodine ablation is contraindicated during pregnancy as it causes increased radiation risk to
the fetus.
o More common in females,
o Between the ages 3rd and 4th decade
o It has rich intrathyroidal lymphatic network and hence higher chance of multifocal
involvement in the thyroid gland
o It has higher incidence of nodal metastases
o Generally, thyroid cancers have nodal metastases in the pretracheal Compartment, i.e. Level
VI.
o Mostly papillary carcinoma presents as a solitary nodule, which is firm in consistency.
o It can cause hoarseness by invasion of recurrent laryngeal nerve.
o USG neck and thyroid shows an irregular hypoechogenic solid lesion with rich intranodal
vascularity and microcalcifications.
o The lesion will be taller than wide on transverse view of the sonography probe.
o The diagnosis is easily established by doing USG guided FNA of the suspicious nodule, which
shows pseudonuclear inclusions, nuclear ingrooving giving an Orphan Annie Eye appearance.
There are also psammoma bodies which are concretions of dead and necrotic tissue that present
with dystrophic calcification.
o Once the diagnosis of papillary carcinoma has been made, patient should have USG neck to
rule out any nodal metastases and look for invasion of nodule to adjacent structures to stage the
disease.
o No need for CECT neck.
o Patient evaluated to have clinically node negative, should undergo total thyroidectomy alone
and those who have central compartment nodes should undergo an additional central
compartmental neck dissection
o Overall papillary carcinoma carries a relatively good prognosis with a 20 year survival rate of
~90%. The 5-year survival is in the range of 95%.
o If the tumor is confined to the gland then mortality is less than 2.5%, with a sharp increase in
mortality in patients with extension beyond the gland, to 38%.

Answer. a

279. A 18-year-old boy presented to the emergency unit with history of fever and abdominal
pain. On examination he was febrile and pulse rate was 104/min,the resident was eliciting the
sign shown below.Identify the sign.

a. Rovsing’s sign

b. Balance sign

c. Blumberg sign

d. Psoas sign

Solution. (c) Blumberg sign


Ref: Read the text below
Sol:

Answer. c
280. A total autogenous latissimus dorsi flap can provide a maximum muscle volume of which
one of the following?

a. 50 – 100 cc

b. 100 – 200 cc

c. 200 – 300 cc

d. 300 – 400 cc

Solution. (d) 300 – 400 cc


• A total autogenous latissimus dorsi flap can provide a maximum muscle volume of 300 – 400
cc,however postoperative muscle atrophy may occur to the tune of 20-25% which should be
taken into consideration.

Answer. d

281. A 56-year-old gentleman came to casualty with history of massive hemoptysis. His chest
Xray and other investigations are normal. Which of the following you will not do to prevent
hemoptysis?

a. Pulmonary artery embolization

b. Bronchial artery embolization

c. Surgical removal of lobe

d. Procoagulant drugs

Solution. (d) Procoagulant drugs


Answer. d

282. Pancreatic cancer has the highest association with:

a. Hereditary pancreatitis

b. Peutz-jeghers syndrome

c. FAP

d. FAMMM syndrome

Solution. (b) Peutz-jeghers syndrome

Answer. b

283. What is the percentage that USG can detect lymph node metastases in a patient with
breast cancer and clinically negative lymph nodes?

a. 10-20%

b. 20-30%

c. 30-50%

d. 50-60%

Solution. (d) 50-60%

Answer. d

284. All except one are true for Sclerosing Encapsulating Peritonitis

a. More common in young females

b. May be commonly associated with tuberculosis

c. Steroids are the 1st line management

d. Presents as abdominal cocoon

Solution. Steroids are the 1st line management


Abdominal cocoon or sclerosing encapsulating peritonitis is a rare cause of intestinal obstruction
that has been described mostly in young adolescent girls.
Thick fibrotic peritoneum encasing the small bowel in a small volume is a pathognomonic
feature.
This condition presents many difficulties in preoperative diagnosis. Early diagnosis can result in
proper management, and may prevent the need for bowel resection.
Intestinal obstruction secondary to abdominal cocoon or peritonitis chronica fibrosa incapsulata
is an infrequent clinical entity
Sclerosing encapsulating peritonitis (SEP) is an acquired condition.
Prior abdominal surgery or peritonitis, chronic ambulatory peritoneal dialysis and prolonged use
of practolol are the main causative factors.
Other conditions such as history of ventriculo peritoneal and peritoneovenous shunts,
sarcoidosis, cirrhosis, systemic lupus erythematosus, propranolol therapy for constrictive
pericarditis, fibroid uterus, endometrioma or tumor of ovary, and recurrent peritonitis have also
been implicated.
Answer. c

285. Measurement of intra-vascular pressure by a pulmonary artery catheter is done in which


phase of respiration?

a. At the end of expiration

b. At peak of inspiration

c. During mid inspiration

d. During mid expiration

Solution. (a) At the end of expiration


• To eliminate the effects of intrathoraic pressure on pulmonary capillary wedge pressure
values, atrial pressure is measured at the end expiration during which there is no movement of
air in and out of thorax.

Answer. a

286. The inframammary crease is better preserved in which of the following TRAM flap?

a. Ipsilateral TRAM flap reconstruction

b. Contralateral TRAM flap reconstruction

c. Bipedicled TRAM

d. All of the above

Solution. (a) Ipsilateral TRAM flap reconstruction


• The inframammary crease is better preserved in ipsilateral pedicle TRAM flap owing to a
direct 180 degree transposition over the costal margin.
• In contralateral TRAM flaps, the pedicle has to be tunneled and passed over the costal margin
and hence it disturbs the definition of the inframammary fold and presents as a bulge when the
patients sits up.

Answer. a

287. Which of the following is regarded as the Management of Choice in a case of rectal
carcinoma at 5 cm from the anal verge?
a. Anterior resection

b. Abdominal perineal resection

c. Posterior resection

d. Local resection

Solution. (b) Abdominal perineal resection


• The Abdominoperineal resection is indicated in the treatment of a rectal cancer in the lower
rectum (0-5cm from the anal verge).
• It includes removal of the rectum, anus, sphincter muscles and a permanent colostomy is
reserved most of the time for patient whose sphincter muscles are involved by tumor or for
patients with rectal cancer who have poor sphincter function preoperatively. Ideally, a surgical
margin of at least 2cm should be obtained.
• Recent evidence indicates that a margin as small as 1cm may be sufficient, thus allowing more
sphincter-saving procedures.
• It is not performed in the presence of peritoneal seeding or fixation to bony pelvis.

Answer. b

288. A young patient presents with a massive injury to proximal duodenum, head of pancreas
and distal common bile duct. The procedure of choice in this patient should be:

a. Roux-en-Y anastomosis

b. Pancreaticoduodenectomy (Whipple's operation)

c. Lateral tube jejunostomy

d. Retrograde jejunostomy

Solution. (b) Pancreaticoduodenectomy (Whipple's operation)


• A Reconstruction surgery referred to as 'Whipple's operation' is required in conditions
characterized by massive injuries of the proximal duodenum and head of pancreas along with
destruction of ampulla and proximal pancreatic duct or distal common bile due to any tumors

Answer. b

289. A male aged 72 underwent routine medical examination. Digital rectal examination shows
a nodule on prostate. Serum examination shows elevated PSA levels. The next step of
investigation will be:

a. CT scan pelvis

b. IVP

c. Prostatic massage for examination of secretions

d. Transrectal ultrasound-guided prostate biopsy

Solution. (d) Transrectal ultrasound-guided prostate biopsy


The appropriate screening examinations for prostate cancer include a serum PSA and DRE. On
the basis of an elevated PSA or an abnormal DRE, the patient should be referred for a
transrectal ultrasound-guided prostate biopsy.

In general, six to twelve cores are taken for evaluation and the areas that are abnormal on DRE
may receive more biopsy attempts.
Also Know:
• Prostate specific antigen (PSA) is a protein produced and secreted by both normal prostate
and prostate cancer cells.
• It provides a sensitive but not highly specific screening test, as it is also elevated with BPH,
inflammation, and infection of the prostate.
• The proposed cutoff is 4.0 ng/mL, but studies have shown that 15% of men with normal PSA
levels will have prostate cancer and 2% will have high-grade prostate cancer.

Answer. d

290. After an alcoholic binge a 50 yrs old person had an episode of vomiting followed by
hematemesis. Endoscopic picture is given below. Which is the best treatment for this patient?

a. Conservative treatment

b. Endoscopic sclerotherapy

c. Band ligation

d. Immediate proximal gastrectomy

Solution. (a) Conservative treatment


• Mallory-Weiss syndrome (occurs after persistent retching) is characterized by upper GI
bleeding due to a mucosal tear (not perforation) below the esophago-gastric junction.
• Forceful vomiting may lead to either disease, but in Boerhaave's syndrome, the tear extends
through all layers of the esophageal wall and in Mallory-Weiss syndrome it is partial thickness
tear.
• Mallory Weiss tear is generally below GE junction, at lesser curvature, presents with
hematemesis, diagnosed by endoscopy and treated conservatively as >95% tear heal
spontaneously.

Answer. a

291. A 4 week old boy presented with non-bilious projectile vomiting. USG reveals a thickened
pylorus of length 19mm and 5mm thickness. What electrolyte abnormality is most probable in
this patient?

a. Hypokalemic, hypochloremic metabolic alkalosis

b. Hyperkalemic, hypochloremic metabolic alkalosis

c. Hyperkalemic, hyperchloremic metabolic alkalosis

d. Hypokalemic, hyperchloremic metabolic alkalosis

Solution. (a) Hypokalemic, hypochloremic metabolic alkalosis


• Infantile Hypertrophic Pyloric Stenosis (IHPS) IHPS classically presents with the gradual onset
of nonbilious projectile vomiting at about the third week of life.
• The typical infant presents with nonbilious projectile vomiting and dehydration (with
hypochloremic hypokalemic metabolic alkalosis).
• The primary symptom is regurgitation progressing to nonbilious, projectile vomiting, which
occurs intermittently or after feeding.

Answer. a

292. Injury to posterior sector duct will lead to damage of biliary drainage of which lobes?

a. V and VIII

b. V and VI

c. VI and VII

d. VII and VIII

Solution. (c) VI and VII

Answer. c
293. Which of the following is not a reversible bariatric surgery?

a. Band gastroplasty (LABG)

b. Gastric balloon

c. POSE (Primary obesity surgery endoluminal)

d. Sleeve gastrectomy

Solution. (d) Sleeve gastrectomy

Answer. d

294. TRISS includes ?

a. RTS+AGE+GCS

b. RTS +ISS+ AGE

c. ISS+GCS+AGE

d. ISS+GCS+HR

Solution. (B) RTS +ISS+ AGE


TRAUMA SCORES:
A. GCS: GLASGOW’S COMA SCALE
B. RTS : REVISED TRAUMA SCALE ( RR+ GCS+SBP)
C. AIS : ABRREVIATED INJURY SCORE( ANATOMY BASED SCORE)
D. TRISS: TRAUMA AD REVISED INJURY SEVERITY SCORE
( RTS+ ISS+AGE+MODE OF INJURY)
E. MESS( MANGLED EXTREMITY SEVERITY SCORE)
INCLUDES: SHOCK+AGE+LIMB ISCHEMIA+ENERGY OF IMPACT)

Answer. b

295. Radical neck dissection includes all except?

a. Cervical node level VI


b. Sternocleidomastoid

c. Spinal accessory nerve

d. Internal jugular vein

Solution. (a) Cervical node level VI

Answer. a

296. Which of the following is not used for lip reconstruction?

a. Abbey Estlander flaps

b. Gillies flap

c. Karapandzic flap

d. Tessier operation

Solution. (d) Tessier operation


.
• Tessier flap is used for cleft palate/face
• The Tessier flap has been defined for the reconstruction of total or subtotal defects of the
lower eyelid.
Answer. d

297. 42 year male admitted with blunt trauma abdomen with suspected renal injury . Identify
the grade of injury mentioned in the image :

a. Grade I

b. Grade V

c. Grade III

d. Grade IV

Solution. d- Grade IV
Answer. d

298. identify the procedure shown in the image :


a. Kocher’s maneuver

b. Pringle’s maneuver

c. Cattle Braasch Maneuver

d. Mattox Maneuver

Solution. A: Kocher’s maneuver

Answer. a

299. Identify the following procedure for benign prostatic hyperplasia:

a. TULIP

b. PUL

c. TPUL

d. TURP
Solution. b : PUL
PUL(Prostatic Urethral Lift; also known as Urolift) .
Transprostatic placement of spring loaded T shaped sutures causing mechanical opening of
prostatic urethra

Answer. b

300. A 26 years old pregnant female presents with sudden onset pain with bluish discoloration
of the left lower extremity. Image below shows the current patient status .All except one are
true:

a. Thrombosis of deep venous system

b. Collaterals are involved

c. Intravenous anticoagulants are 1st line of management

d. Duplex imaging is a useful diagnostic tool

Solution. B: Collaterals are involved


Condition shown below is Plegmesia Cerulea Dolens, where there is extensive thrombosis of the
deep axial venous system with sparing of the collaterals.This is also known as Blue Limb
Plegmesia alba dolens (white limb ) involves axial venous system with extensive involvement .
Pain, tenderness over the limb is common presentation.
Duplex is imaging modality of choice
Intravenous anticoagulants are the 1st  line management.
Target INR is 2-3.5 , if INR stays >2 for more than 24 hrs, oral warfarin is added.
Oral warfarin is continued at least for 3 months .

Answer. b

Test Answer

1. (d) 2. (b) 3. (b) 4. (b) 5. (d) 6. (b) 7. (b) 8. (a)


11. (c) 12. (c) 13. (d) 14. (a) 15. (c) 16. (d) 17. (c) 18. (c)
21. (a) 22. (c) 23. (c) 24. (d) 25. (c) 26. (b) 27. (a) 28. (d)
31. (d) 32. (d) 33. (d) 34. (d) 35. (a) 36. (b) 37. (b) 38. (c)
41. (c) 42. (c) 43. (b) 44. (d) 45. (a) 46. (a) 47. (d) 48. (b)
51. (b) 52. (a) 53. (a) 54. (b) 55. (b) 56. (b) 57. (a) 58. (d)
61. (c) 62. (d) 63. (c) 64. (d) 65. (c) 66. (b) 67. (d) 68. (b)
71. (b) 72. (b) 73. (b) 74. (b) 75. (b) 76. (b) 77. (b) 78. (d)
81. (c) 82. (c) 83. (d) 84. (a) 85. (a) 86. (a) 87. (d) 88. (a)
91. (b) 92. (c) 93. (c) 94. (a) 95. (d) 96. (c) 97. (b) 98. (b)
101. (d) 102. (d) 103. (a) 104. (a) 105. (b) 106. (b) 107. (c) 108. (c)
111. (d) 112. (d) 113. (b) 114. (a) 115. (a) 116. (c) 117. (c) 118. (b)
121. (b) 122. (d) 123. (d) 124. (a) 125. (a) 126. (b) 127. (c) 128. (c)
131. (c) 132. (d) 133. (c) 134. (b) 135. (b) 136. (b) 137. (a) 138. (c)
141. (d) 142. (c) 143. (b) 144. (c) 145. (c) 146. (b) 147. (c) 148. (b)
151. (b) 152. (d) 153. (b) 154. (c) 155. (b) 156. (d) 157. (b) 158. (b)
161. (c) 162. (a) 163. (a) 164. (c) 165. (c) 166. (c) 167. (c) 168. (a)
171. (c) 172. (b) 173. (d) 174. (c) 175. (d) 176. (c) 177. (c) 178. (c)
181. (a) 182. (b) 183. (c) 184. (c) 185. (b) 186. (c) 187. (b) 188. (c)
191. (b) 192. (d) 193. (c) 194. (d) 195. (d) 196. (a) 197. (b) 198. (d)
201. (d) 202. (b) 203. (d) 204. (b) 205. (b) 206. (d) 207. (c) 208. (a)
211. (a) 212. (b) 213. (c) 214. (a) 215. (a) 216. (b) 217. (d) 218. (a)
221. (c) 222. (d) 223. (d) 224. (c) 225. (d) 226. (a) 227. (d) 228. (c)
231. (c) 232. (a) 233. (b) 234. (c) 235. (a) 236. (d) 237. (a) 238. (c)
241. (d) 242. (c) 243. (b) 244. (a) 245. (a) 246. (b) 247. (d) 248. (c)
251. (c) 252. (d) 253. (a) 254. (b) 255. (d) 256. (a) 257. (b) 258. (c)
261. (a) 262. (a) 263. (c) 264. (a) 265. (b) 266. (c) 267. (d) 268. (c)
271. (b) 272. (c) 273. (a) 274. (b) 275. (d) 276. (a) 277. (c) 278. (a)
281. (d) 282. (b) 283. (d) 284. (c) 285. (a) 286. (a) 287. (b) 288. (b)
291. (a) 292. (c) 293. (d) 294. (b) 295. (a) 296. (d) 297. (d) 298. (a)

Vous aimerez peut-être aussi